MSK Flashcards
A 32-year-old footballer sustains an avulsion injury to the anterior superior iliac spine during training. Which of the following muscles is likely to be affected? [B1 Q12]
a. Sartorius
b. Gracilis
c. Iliopsoas
d. Rectus femoris
e. Semimembranosus
Sartorius
Sartorius has its origin at the anterior superior iliac spine and inserts into the pes anserinus. A sartorius muscle injury can therefore cause an avulsion fracture of the anterior superior iliac spine. Gracilis has its origin at the inferior pubic ramus, and rectus femoris has its origin at the anterior inferior iliac spine.
Other avulsion sites and tendons
Rectus femoris – AIIS
All hamstrings – Ischial tuberosity
Iliopsoas – Lesser trochanter
Gluteus medius and minimus – Greater trochanter
Transversus abdominis – Iliac crest
A 13-year-old footballer complains of pain in the right groin after a tackle. Radiograph of the pelvis shows an avulsion fracture of the lesser tuberosity. Which muscle is attached to the lesser tuberosity? [B5 Q25]
a. Iliopsoas
b. Rectus femoris
c. Sartorius
d. Biceps femoris
e. Vastus medialis
Iliopsoas muscle
This is attached to the lesser trochanter.
A 52-year-old woman presents to her GP with a longstanding history of lower back pain which has suddenly worsened in severity over the past few days. An urgent MRI scan of the lumbar spine shows a right paracentral disc protrusion at the L4/L5 level. The disc impinges on the lateral recess at this level. The most likely nerve to be affected is the: [B2 Q26]
a. Cauda equina
b. Lumbar plexus
c. Right L4
d. Right L5
e. Right S1
Right L5
The right L5 nerve root is the most likely to be affected as it will be sitting in the right lateral recess at the L4/5 level. The L4 nerve root will be at the exit foramen and therefore if the protrusion affects only the lateral recess, then this nerve will already have exited and therefore not be affected.
An MRI of the ankle shows deep injury to the deltoid ligament. Which of the following belong to the deep components of the deltoid (medial collateral) ligament of the ankle? [B2 Q22]
a. Tibio-calcaneal ligament
b. Tibio-navicular ligament
c. Posterior superficial tibiotalar ligament
d. Anterior tibiotalar ligament (ATTL)
e. Tibio-spring ligament
D
The other answers all belong to the superficial components. The posterior deep tibiotalar ligament is also deep. The superficial and deep components function almost synergistically and stabilise against valgus and pronation as well as rotational force against the talus.
A 56-year-old woman slips off the pavement onto the road and her outstretched foot is run over by a passing car. She has immediate severe midfoot pain. Plain radiographs taken on arrival at the emergency department confirm a Lisfranc fracture dislocation of the midfoot. Which two bones does the Lisfranc ligament attach to? [B2 Q36]
a. First metatarsal and intermediate cuneiform
b. First metatarsal and medial cuneiform
c. Second metatarsal and medial cuneiform
d. Second metatarsal and intermediate cuneiform
e. First and second metatarsals to the medial and intermediate cuneiforms
Second metatarsal and medial cuneiform
The Lisfranc ligament attaches between the second metatarsal and medial cuneiform, which is why an injury to this ligament allows the second to fifth metatarsals to drift laterally once they have lost this stabilisation. This is therefore an unstable injury and requires rapid immobilisation. This is a vital injury to detect as long-term sequelae will often result from a delayed diagnosis.
Of the lateral fibrous structures contributing to the stability of the posterolateral corner of the knee, which is the most likely to be congenitally absent and not identified on MRI, being present in only approximately two-thirds of patients? [B4 Q6]
a. Lateral collateral ligament
b. Popliteus tendon
c. Popliteo-fibular ligament
d. Arcuate ligament
e. Fabello-fibular ligament
Arcuate ligament
The structures of the posterolateral corner of the knee have a very important role in maintaining the rotational stability of the knee joint.
The lateral collateral ligament forms the superficial layer, with the remainder of the structures comprising the deep layer. Injury is relatively common and results most frequently from a varus force on an extended joint.
The lateral collateral ligament and popliteus tendon are present in all joints, with the popliteofibular ligament being present in approximately 98%.
Both the arcuate ligament and fabellofibular ligaments are variable, with the former absent more frequently. Absence of one of these structures is often compensated for by hypertrophy of the other.
Following wrist arthrography by a single-compartment radiocarpal injection technique, contrast seen on MR arthrographic images in the midcarpal compartment can be explained by disruption of which of the following structures? [B4 Q98]
a. triangular fibrocartilage
b. lunotriquetral ligament
c. dorsal distal radioulnar ligament
d. flexor retinaculum
e. radio scapholunate ligament
Lunotriquetral ligament
The two most important intercarpal ligaments are the scapholunate and lunotriquetral ligaments. These are crescent-shaped with strong anterior and posterior zones and a relatively thin middle membrane.
Disruption of Scapholunate or lunotriquetral ligaments will result in communication of the radiocarpal compartment proximally with the midcarpal compartment distally.
Contrast material seen in the distal radioulnar joint indicates disruption to the triangular fibrocartilage complex or distal radioulnar ligaments.
Some authors advocate selective midcarpal injection as superior in delineating injury to the scapholunate and lunotriquetral ligaments, and a sequential technique of three injections has also been described.
A 20-year-old football player presents after injuring his right knee in a tackle. Plain radiographs show fracture of the tibial spine with lipohaemarthrosis. What structure is attached to the medial part of the anterior tibial spine? [B5 Q36]
a. Anterior cruciate ligament
b. Posterior cruciate ligament
c. Medial collateral ligament
d. Lateral collateral ligament
e. Medial meniscus
Anterior cruciate ligament
The anterior cruciate ligament is attached to the medial part of the tibial spine.
A 60-year-old presents with left groin pain. Ultrasound shows a 2 cm hypoechoic lesion bulging medial to the epigastric vessels on Valsalva manoeuvre and absent on rest. What is the most likely diagnosis? [B5 Q38]
a. Direct inguinal hernia
b. Indirect inguinal hernia
c. Obturator hernia
d. Spigelian hernia
e. Femoral hernia
Direct inguinal hernia
A direct inguinal hernia is seen medial to the inferior epigastric vessels whereas an indirect hernia is seen lateral to them.
A 40-year-old man presents with right groin pain. Ultrasound shows a 3 cm echogenic soft tissue mass distending the right inguinal canal on straining, and which goes away on relaxation. What is the most likely diagnosis? [B5 Q39]
a. Direct inguinal hernia
b. Indirect inguinal hernia
c. Femoral hernia
d. Obturator hernia
e. Lymph node
Indirect inguinal hernia
An indirect inguinal hernia protrudes through the internal inguinal ring and extends along the inguinal canal parallel to its long axis.
An 18-year-old football player presents with right groin pain after a tackle. The radiograph shows avulsion of the lesser trochanter. Which muscle is attached to the lesser trochanter? [B5 Q42]
a. Sartorius
b. Rectus femoris
c. Iliopsoas
d. Hamstrings
e. Adductor longus
Iliopsoas
Iliopsoas is attached to the lesser trochanter.
An 18-year-old male with fingernail dysplasia and a family history of renal failure is investigated for possible nail-patella syndrome. Which of the following radiographic findings is considered pathognomonic for this disorder? [B1 Q54]
a. Patellar hypoplasia
b. Lateral elbow hypoplasia
c. Posterior iliac horns
d. Calcaneo-valgus feet
e. Madelung deformity
Posterior iliac horns
Nail-patella syndrome (Fong Disease) (hereditary onycho-osteo-dysplasia) is an autosomal dominant condition characterized by:
Nail dysplasia
Patella hypoplasia
Elbow hypoplasia
Iliac horns present in 80% and are pathognomic.
They arise at the site of gluteus medius and project posterolaterally. Patella hypoplasia results in chronic knee pain and recurrent dislocations. Elbow hypoplasia is typically towards the lateral side of the joint. Madelung deformity and calcaneo-valgus feet are other features described in nail-patella syndrome. The most important non-orthopaedic condition is an immune complex nephropathy, which can result in end-stage renal failure. These patients are also at risk of open-angle glaucoma.
Fong Disease (Nail Patella Syndrome) [STATdx]
Synonyms:
- Hereditary Osteo-onycho-dysplasia
- Iliac Horn Syndrome
- Turner-Kieser syndrome
Pathology: Autosomal Dominance
Clinical Features:
- Knee pain and instability
- Characteristic Nail Deformities – abnormal/ triangular lunula, discoloured, pitting
ridged, thickened nails, starts from thumb to little finger, worse on ulna side. - Kidneys – proteinuria to nephrotic syndrome.
- Open angle glaucoma – peripheral vision loss.
Radiographic Features:
- Iliac horns – Pathognomonic, 80% of cases
Symmetric, posterior-laterally from central ilium,
Asymptomatic
Palpable 2. Absent/hypoplastic patella
Asymmetric
Superolateral dislocation
Knees appear flattened
Overgrowth of medial femoral condyle
Hypoplastic lateral femoral condyle
Enlarged tibial tubercle - Absent/hypoplastic radial head
Asymmetric
Radial head may subluxate or dislocate
Hypoplastic capitellum and lateral condyle
Prominent medial condyle
Creates positive ulnar variance deformity at wrist - Cubitus valgus
Limited motion - Others
Hypoplastic changes of shoulder, hip
Talipes equinovarus and other foot deformities*
A 30-year-old woman presents to her general practitioner with fatigue and painful stiff knees.
She is subsequently found to be anaemic. Plain films show an Erlenmeyer flask deformity of
the distal femora with cortical thinning. There are no erosions. What is the most likely
underlying condition? [B4 Q2]
a. mucopolysaccharidosis
b. rheumatoid arthritis
c. Gaucher’s disease
d. Langerhans’ cell histiocytosis
e. thalassaemia major
Gaucher’s disease
Gaucher’s disease is the most common lysosomal storage disorder with an incidence of 1:50
000 (100 times more common in Ashkenazi Jews). It is caused by a defect of hydrolase acid b-
glycosidase, which results in accumulation of the fatty substance glucosylceremide within
macrophages in the reticuloendothelial system. It characteristically causes an Erlenmeyer flask
deformity of the distal femur or proximal tibia due to marrow infiltration. Patients may be
asymptomatic or present with anaemia, large joint stiffness or bone pain. Diagnosis is by bone
marrow aspirate. The mucopolysaccharidoses are a spectrum of lysosomal storage diseases that
typically present in infancy with a variety of overt symptoms and signs. Rheumatoid arthritis
can present with anaemia and joint stiffness, but marrow infiltration is not a feature on plain
film. Musculoskeletal manifestations of Langerhans’ cell histiocytosis most commonly affect
the skull (50%). Although Erlenmeyer flask deformity is seen in thalassaemia major,
presentation is within the first 2 years of life.
A 10-year-old white boy presented with mass in abdomen and bilateral hip pain. He was found to have splenomegaly and pancytopaenia. Pelvic radiograph suggests bilateral avascular necrosis of femoral heads. MRI shows diffuse low signal bone marrow on T1 and T2. What is the most likely diagnosis? [B5 Q11]
(a) Gaucher’s disease
(b) Sickle cell disease
(c) Perthes’ disease
(d) Leukaemia
(e) Multifocal histiocytosis
Gaucher’s disease
This is the commonest lipid storage disorder. Marrow infiltration leads to avascular necrosis in the femur, ankle, and humerus. Patients have splenomegaly, anaemia, and pancytopenia. Loss of normal remodelling of the femur results in Erlenmeyer flask’ deformity at the distal femur.
A 10-year-old male involved in an RTA is brought to the A&E department with a history of
severe right thigh pain. Plain radiograph demonstrates a transverse fracture in the mid-
diaphysis of the femur. Incidental note is made of bone osteopenia and undertubulation of the
femur with metaphyseal flaring producing Erlenmeyer flask deformity and coxa magna related
to previous avascular necrosis of the femoral head. What is the underlying bone disease? [B1
Q55]
A. Pyle’s disease.
B. Osteopetrosis.
C. Gaucher’s disease.
D. Fibrous dysplasia.
E. Ollier’s disease.
Gaucher’s disease.
All these conditions cause Erlenmeyer flask deformity and are associated with pathological
fractures. However, the history of previous avascular necrosis of femoral head suggests
Gaucher’s disease. Sickle-cell disease (SCD) may also cause all the above bone changes.
Gaucher’s disease is a rare familial metabolic disorder caused by deficiency of the enzyme F-
glucocerebrosidase. This leads to accumulation of glucocerebroside in reticuloendothelial cells
(macrophages) of the liver, spleen, and bone marrow.
The imaging findings include delayed growth, osteopenia, Erlenmeyer flask deformity,
metaphyseal notching of humeri, bone infarction/avascular necrosis, and pathological fractures.
Diffuse marrow replacement with low signal on T1WI is noted on MRI. Visceral
manifestations include hepatosplenomegaly and reticulonodular interstitial lung disease.
Gaucher’ Disease [STATdx]
Pathology: – Autosomal Recessive, Glucocerebrosidase deficiency, deposition in RE cells
General Features:
Organomegaly and pancytopenia, relative sparing of hepatocytes, low incidence of liver failure.
Acute bony crisis secondary to infarct
Other skeletal manifestation Erlenmeyer flask deformity of distal femora, pathological
fractures, vertebral collapse, and bony infarct.
Radiographic Findings:
* Vertebral endplate collapse, femoral Erlenmeyer flask deformity, avascular necrosis
(AVN), pathologic fractures
* Hepatosplenomegaly
* MRI Findings
o Early hip AVN
o Reduced marrow signal due to glucocerebroside deposition
o “Ballooning” of intervertebral discs due to vertebral endplate collapse
* Nuclear Med – “Cold” lesion in bone infarcts
* Best imaging tool
o Plain radiographs to characterize disease extent
o MR for early bone infarct detection
A 34-year-old man is admitted with sudden onset chest pain described as tearing in nature.
Clinical examination reveals a diastolic murmur consistent with aortic regurgitation.
Subsequent chest CT confirms ascending aortic dissection. He has a past medical history of
spontaneous pneumothorax. Despite a negative family history, an underlying diagnosis of
Marfan syndrome is suspected. Which of the following musculoskeletal manifestations is
required for this diagnosis to be made? [B1 Q59]
A. Joint hypermobility.
B. Pectus excavatum of moderate severity.
C. Reduced upper-to-lower segment ratio.
D. High arched palate.
E. Malar hypoplasia.
Reduced upper-to-lower segment ratio.
Marfan syndrome
* An autosomal dominant, multisystem connective tissue disorder.
* Approximately 25% are sporadic mutations.
* Mutation of the fibrillin-1 gene.
* There is a broad phenotype expression – diagnosis can be made with the Ghent
classification system.
* In the absence of a family history, the presence of two major criteria in two different organ
systems and a minor criterion in a third system supports a diagnosis of Marfan syndrome.
In this case, dissection of the ascending aorta is a major cardiovascular criterion and
spontaneous pneumothorax a minor pulmonary system criterion. Of the musculoskeletal
manifestations, reduced upper-to-lower segment ratio is a major criterion, the remaining
options are all minor criteria. Other musculoskeletal system major criteria include scoliosis
with a curvature greater than 20’, pectus carinatum, pectus excavatum requiring surgery,
acetabular protrusion, and medial displacement of the medial malleolus causing pes planus.
A 30-year-old African man presents with knee pain. Radiograph shows a serpiginous area of
lucency with sclerotic margins in the proximal metaphysis of tibia. MRI shows a ‘double-line’
sign on T2-weighted images. There is a linear area of low signal peripheral to a high signal
intensity inner border. A bone scan shows no uptake in the area. The most likely diagnosis is?
[B5 Q27]
(a) Bone infarct
(b) Osteomyelitis
(c) Enchondroma
(d) Non-ossifying fibroma
(e) Osteonecrosis
Bone infarct
The radiographic and MRI appearances described are typical for a bone infarct. These are
typically metaphyseal or diaphyseal in contrast to osteonecrosis. Bone scans may show
increased uptake in acute stages where revascularisation has occurred.
A 5-year-old boy presents with a history of walking difficulty. On examination he is noted to
have an antalgic gait and lower limb length discrepancy, with the right limb being shorter than
the left. Plain radiographs of the right leg show lobular ossific masses arising from the distal
femoral epiphysis and the talus, which resemble osteochondromas. What is the most likely
underlying diagnosis? [B1 Q66]
A. Dysplasia epiphysealis hemimelica (Trevor disease).
B. Multiple epiphyseal dysplasia.
C. Diaphyseal aclasis.
D. Dyschondrosteosis (Leri–Weil disease).
E. Klippel–Trenaunay–Weber syndrome.
Dysplasia epiphysealis hemimelica (Trevor disease).
This is an uncommon developmental disorder relating to the formation of an osteochondroma
type lesion at the epiphyses of usually a single lower extremity. The epiphyses most involved
are those on either side of the knee or ankle. Typically, it is only the medial or lateral side of
the epiphyses affected (medial: lateral 2:1). The disease is usually recognized at a young age
because of an antalgic gait, palpable mass, varus or valgus deformity, or limb length
discrepancy.
Dysplasia epiphysealis hemimelica – Trevor disease [Radiopaedia]
Pathology: Non-hereditary, Rare, Male Predilection.
Clinical Features: Young Children, Walking difficulty, Antalgic Gait, Discrepancy in leg
lengths
Radiological Features:
Osteochondroma arising from epiphyses, so epiphyseal signal. Widening of Joint Space.
Lower > Upper,
Distal > Proximal,
Medial > Lateral
Dysplasia epiphysealis multiplex – Fairbank disease [Radiopaedia]
Autosomal dominance
Delayed, irregular secondary ossification centres,
Flatting and squaring of epiphyses, eg. Thinning or lateral tibial epiphysis
Double-layered patella
Hypoplastic tibial and femoral condyles with shallow intercondylar notch
Hereditary Multiple Exostosis – Diaphyseal Aclasis [Radiopaedia]
Autosomal dominance
Multiple osteochondroma – typical appearance of osteochondroma.
Can be asymptomatic small lesions or large lesion with deformity.
Any bone except calvarium.
An orthopaedic surgeon in your hospital comes to your office to ask your advice on a 15-year-
old girl he is about to see at his clinic. Although limited clinical information is available, he
was able to find out that the patient has a congenital condition, which has resulted in her being
confined to a wheelchair. As she was complaining of a sore knee, an x-ray was carried out.
There is a long gracile femur and tibia, indicating under-tubulation of the bone. What is the
most likely cause for this appearance? [B1 Q70]
A. Dwarfism.
B. Gaucher’s disease.
C. Cerebral palsy.
D. Arthrogryposis multiplex congenital.
E. Juvenile RA (JRA).
Cerebral palsy.
As a radiologist you would obviously have been able to correct your orthopaedic colleague,
that long gracile bones are examples of over tubulation, not under tubulation. As such options
A, B, and E are not diagnostic considerations as these result in under tubulation and may cause
an Erlenmeyer flask abnormality. This phenomenon is further described elsewhere in this
chapter but causes of Erlenmeyer flask abnormality include anaemias (thalassaemia, SCD),
storage disorders (Gaucher’s, Niemann–Pick), and skeletal dysplasias (Pyle’s disease,
craniometaphyseal dysplasia, Melnick–Needles syndrome).
The most common cause of over-tubulation is in patients with diminished weight bearing
(cerebral palsy, myelomeningocele, arthrogryposis), with cerebral palsy being the most
common of these. JRA and Marfan syndrome can also cause this appearance.
Bone Tubulation [Radiopaedia]
the adult-type appearance of long bones with a diaphyseal narrowing or constriction that forms
due to periosteal bone resorption and endosteal bone formation at the metaphysis during
longitudinal bone growth at the physis.
Causes of over-tubulation
1. Osteogenesis imperfecta
2. Neurofibromatosis
3. Paralysis
4. Radiation therapy
Causes of under-tubulation [Erlenmeyar flask deformity]
1. Gaucher
2. Thalassaemia
3. Cranio-metaphyseal Dysplasia
4. Hereditary multiple exostosis
A 21-year-old patient attends the A&E department following a minor injury with a suspected
fracture. The request form states that the patient has osteogenesis imperfecta. It is noted that
the patient is of reduced stature and does not display any evidence of blue sclera, but that the
colouration of his sclera has faded over time. He has normal hearing. What subtype of
osteogenesis imperfecta does he likely have? [B1 Q72]
A. Type I.
B. Type II.
C. Type III.
D. Type IV.
E. Type V.
Type IV.
Osteogenesis imperfecta in an adult is almost always type I or IV. Type I is the most common.
Patients have can have normal stature and the characteristic blue sclera are seen in 90%.
Patients also often have hearing impairment. Type IV has variable bone fragility, from mild to
severe. Hearing impairment is less common, as is reduced stature. Blue sclerae are present in
children but are often absent after adolescence. Type II is universally fatal in the neonatal
period. Type III is also severe and often associated with reduced lifespan. Stature is
significantly reduced. In patients who survive to adolescence the blue sclera are also often
absent. Type V is not universally recognized but is like type IV.
Type IV.
Osteogenesis imperfecta in an adult is almost always type I or IV. Type I is the most common.
Patients have can have normal stature and the characteristic blue sclera are seen in 90%.
Patients also often have hearing impairment. Type IV has variable bone fragility, from mild to
severe. Hearing impairment is less common, as is reduced stature. Blue sclerae are present in
children but are often absent after adolescence. Type II is universally fatal in the neonatal
period. Type III is also severe and often associated with reduced lifespan. Stature is
significantly reduced. In patients who survive to adolescence the blue sclera are also often
absent. Type V is not universally recognized but is like type IV.
You are reviewing the x-rays of a child that are stored in your department’s museum.
Sequential radiographs have been taken as the child has aged and the appearances have become
more pronounced with time. The child has a form of dwarfism. On the CXR you notice ‘oar-
shaped’ ribs. The metacarpals are short and wide, but narrow proximally, giving a fan-like
appearance. The patient has a J-shaped sella turcica. The iliac wings are wide, but the iliac
bones narrow inferiorly. On the lateral lumbar spine, the vertebrae have central anterior beaks.
A clinical vignette mentions that the patient was not intellectually impaired. What condition
does the patient probably have? [B1 Q74]
A. Campomelic dysplasia.
B. Niemann–Pick disease.
C. Morquio syndrome.
D. Achondroplasia.
E. Hurler’s syndrome
Morquio Syndrome
The constellation of skeletal manifestations describes the characteristic appearance of dyostosis
multiplex. This pattern of skeletal abnormalities is seen with the mucopolysaccharidoses
(MPS), although it can also be seen with other storage disorders. Except for Hurler syndrome,
where the manifestations are present at 1 year of age, the skeletal manifestations progress as
the patients get older. Hurler’s and Morquio’s are the most common of the MPS conditions.
Amongst the MPS conditions, Morquio’s stands out as a favourite for single best answer (SBA)
and viva questions as it is the only MPS where the patient is not intellectually impaired. It also
displays a central anterior vertebral body beak, whereas the other conditions have an anterior
beak in the lower third of the vertebral body.
A 35-year-old man presents with increasing stiffness in his knee and soft tissue swelling around
the joint. Plain films show multiple areas of irregular cyst-like radiolucency in the distal femur.
There are no areas of abnormal calcification and there is no evidence of periarticular
osteoporosis. MR shows a low signal joint effusion on both T1 and T2 sequences. The most
likely diagnosis is: [B2 Q2]
a. Synovial osteochondromatosis
b. Pigmented villo-nodular synovitis
c. Osteoarthritis
d. Reiter’s syndrome
e. Osteomyelitis
Pigmented villo-nodular synovitis
Pigmented villo-nodular synovitis is a relatively rare condition which usually presents in the third or fourth decade. It is a monoarticular, painful disease which causes a decreased range of movement at the affected joint. It is most common at the knee (80%) followed by the hip, ankle, shoulder, and elbow. Haemorrhagic ‘chocolate’ effusion is characteristic.
Low signal effusion on all sequences at MR is characteristic. There is no calcification or osteoporosis, and joint space narrowing is a late feature.
Pigmented villonodular synovitis (PVNS) [STATdx]
- Synonyms:
o Teno-synovial Giant Cell Tumour, Intra-articular
o Benign synovioma; focal nodular synovitis - Definitions
o Low-grade fibro-histiocytic proliferation with hemosiderin deposits in synovial
joints - 2 subtypes
o Diffuse tenosynovial giant cell tumor (TSGCT)
o Localized TSGCT - Location - Synovial joints
o Knee: 80% of cases (infrapatellar fat pad > suprapatellar recess > posterior
intercondylar notch > adjacent to posterior cruciate ligament)
o Hip, ankle, elbow, shoulder, wrists, fingers, toes - Morphology
o Diffuse TSGCT: widespread distribution corresponding to shape of joint and
associated synovial spaces
o Localized TSGCT: singular round, ovoid, or lobulated mass in synovial joint - Radiographic Findings
o No mineralization
o Effusion/joint distension
o Cartilage preserved until late in process - CT Findings
o May have increased attenuation related to hemosiderin deposition
o Well-defined erosion with sclerotic margins
o Synovium enhances on C+ imaging - MR Findings
o Effusion
o T1: predominantly low to intermediate SI (hemosiderin deposition)
o Internal signal that shows fat is uncommon finding (from lipid-laden
macrophages)
o T2: predominantly low to intermediate SI; (hemosiderin deposition)
o Gradient-echo: assess for blooming (hemosiderin deposition)
o C+: expected to avidly enhance inhomogeneously, but enhancement is not
present in all cases - Ultrasonographic Findings
o Utility for intraarticular mass localization for biopsy
o Confirms solid soft tissue lesion
o Hypoechoic or mixed echogenicity ± hypervascularity - PET
o May demonstrate increased FDG avidity
A 40-year-old man presents with right knee pain. Plain radiography shows a large joint effusion.
MRI of the knee shows multiple foci of low signal intensity seen in the synovium on T1, T2
and gradient-echo sequences. There is a moderate joint effusion. The most likely diagnosis is?
[B5 Q12]
(a) Haemangioma
(b) Pigmented villo-nodular synovitis
(c) Rheumatoid arthritis
(d) Synovial sarcoma
(e) Synovial chondromatosis
Pigmented villonodular synovitis
This is a benign pathology affecting usually the knee joint. It shows no calcifications,
osteoporosis or erosions (until late). MRI is diagnostic, the lesions returning low signal on all
sequences due to iron (haemosiderin)
A young man with limited range of movement at the shoulder joint, a webbed neck and plain
film findings of a hypoplastic scapula which is elevated and medially rotated with an associated
omovertebral bone is likely to have which associated syndrome? [B2 Q26]
a. Turner’s syndrome
b. Down’s syndrome
c. Klippel–Feil syndrome
d. Neurofibromatosis
e. Cleidocranial dysostosis
Klippel–Feil syndrome
The collective findings described are of a Sprengel deformity of the shoulder. This occurs
because of failure of descent of the scapula secondary leading to both cosmetic and functional
impairment. The male: female ratio is 3:1 and it is associated with Klippel–Feil syndrome, a
condition in which there is fusion of vertebral bodies, and renal anomalies.
A 17-year-old patient complains of lower thoracic back pain. Plain radiographs of the thoraco-
lumbar spine show wedging of multiple vertebrae at the thoraco-lumbar junction, multiple
limbus vertebrae, an increase in the AP diameter with a reduction in the sagittal height of
multiple vertebrae, and multiple endplate defects. What is the unifying diagnosis? [B2 Q31]
a. Scheuermann’s disease
b. Ankylosing spondylitis
c. Mycobacterium tuberculosis
d. Hyperparathyroidism
e. DISH
Scheuermann’s disease
These are the classical appearances of Scheuermann’s disease. This condition usually presents at puberty and consists of:
Vertebral wedging
Endplate irregularity
Narrowing of the intervertebral disc spaces
The most common location is in the lower thoracic and upper lumbar spine. Schmorl’s nodes are often present.
A young man presents to his GP complaining of longstanding back pain. He says he has been
diagnosed with a ‘syndrome’ in the past but cannot remember the details. Which of the
following signs is more likely to suggest a diagnosis of homocystinuria than Marfan’s
syndrome? [B2 Q35]
a. Arachnodactyly
b. Osteoporosis
c. Scoliosis of the spine
d. Autosomal dominant inheritance
e. Upward lens dislocation
Osteoporosis
Osteoporosis is a feature of homocystinuria and occurs in 75% of cases, often causing bowing
and fracture of the long bones. The other features are all more suggestive of Marfan’s syndrome.
Although arachnodactyly does occur in homocystinuria (in 30% of cases), it occurs in 100%
of people with Marfan’s syndrome. Homocystinuria has an autosomal recessive mode of
inheritance
A dental radiograph of a 47-year-old woman shows loss of the lamina dura of most of the teeth.
Which of the following would be a possible cause? [B2 Q45]
a. Osteopetrosis
b. Hypoparathyroidism
c. Scleroderma
d. Sickle cell anaemia
e. Myeloma
Scleroderma
The other causes of loss of the lamina dura include Cushing’s disease, Paget’s,
hyperparathyroidism, osteoporosis, osteomalacia, leukaemia, metastases and Langerhans’ cell
histiocytosis. Both osteopetrosis and hypoparathyroidism cause thickening of the lamina dura
of the teeth.
A 70-year-old woman with a history of dysphagia presents with multiple swelling in the hands.
Radiographs of the hands show widespread soft tissue calcification with terminal phalangeal
resorption. What is the most likely diagnosis? [B5 Q47]
(a) Systemic lupus erythematosus
(b) Scleroderma
(c) Dermatomyositis
(d) Psoriasis
(e) Calcium pyrophosphate deposition disease
Scleroderma
Scleroderma is the cutaneous manifestation of progressive systemic sclerosis. This causes
fibrosis and small vessel disease in several organs. In the hands, typically it causes terminal
phalangeal resorption due to pressure atrophy, soft tissue calcification and occasionally intra-
articular calcification.
A three-month-old boy presents with several small painful soft-tissue swellings which have
developed over the mandibular region and the right clavicle. Plain films show marked
periosteal new bone formation and localised soft tissue swelling. There is also bone expansion
with remodelling of old cortex. The most likely diagnosis is: [B2 Q54]
a. Caffey disease
b. Hypervitaminosis A
c. Infantile myofibromatosis
d. Scurvy
e. Kinky hair syndrome
Caffey disease
The most likely diagnosis is Caffey disease. This is a relatively rare self-limiting condition
which usually presents before six months of age. The mandible is the most common site and
accounts for 80% of cases, followed by the clavicle and the upper limb bones.
A three-year-old boy attends A&E with a history of a seizure. He has known congenital
cardiomyopathy. A chest radiograph shows sclerosis and expansion of several ribs. Previous
plain films have shown bone islands within the vertebrae and long bones and bone cysts within
the phalanges. Which of the following conditions would be likely to underly these findings?
[B2 Q59]
a. Down’s syndrome
b. Tuberous sclerosis
c. Sturge–Weber syndrome
d. Neurofibromatosis
e. Sarcoidosis
Tuberous sclerosis
This is a multi-system autosomal dominant disorder affecting the central nervous system, kidneys, lung and heart. The classic triad of facial angiofibroma, epileptic seizures and mental retardation is only seen in approximately 30% of patients.
Skeletal abnormalities in TS include:
sclerotic calvarial patches or ‘bone islands’
Thickening of diploe
Expansion and sclerosis of ribs
Periosteal thickening of long bones.
Gracile ribs are often seen in association with Down’s syndrome.
A 40-year-old man with short stature and normal intelligence has features including a large
head, and an injury to his left leg whilst playing football. On review of the plain film, no acute
bony injury is demonstrated. However, he is noted to have a disproportionately large fibula, a
champagne glass pelvic inlet and horizontal acetabulae. Which is the underlying diagnosis?
[B3 Q14]
A. Achondrogenesis
B. Homozygous achondroplasia
C. Heterozygous achondroplasia
D. Pseudoachondroplasia
E. Metatrophic dwarfism
Heterozygous achondroplasia
Patients with pseudoachondroplasia have a normal skull and medial beaking of the proximal
femoral neck. Achondrogenesis is lethal in utero/neonatal period, as is homozygous
achondroplasia. In metatrophic dwarfism, dumbbell shaped long bones and flattened vertebra
may be found
A 41-year-old male presents to the A&E department with knee pain following a fall at work.
Plain radiography does not demonstrate any fracture, but note is made of continuous, irregular
cortical hyperostosis along the lateral margin of the femur. What is the most likely diagnosis?
[B1 Q68]
A. Osteopoikilosis.
B. Fibrous dysplasia.
C. Engelmann disease.
D. Melorheostosis.
E. Osteopathia striata.
Melorheostosis.
The radiographic findings describe the ‘flowing candle-wax’ sign (continuous, irregular cortical hyperostosis in a long bone), which indicates melorheostosis, a non-hereditary sclerosing bone dysplasia of unknown aetiology. Patients are often asymptomatic, being discovered incidentally. It is most common in the long bones. The disease can overlap with other sclerosing bone dysplasias such as:
Osteopoikilosis (multiple ovoid bone islands)
Osteopathia striata (metaphyseal longitudinal striations).
Engelmann disease presents in childhood with neuromuscular dystrophy. Diaphyseal fusiform enlargement with cortical thickening is seen in the long bones. Fibrous dysplasia causes bone thinning.
Which of the following skeletal findings on plain radiographs is not typically associated with
achondroplasia? [B4 Q21]
a. short interpedicular distance
b. small foramen magnum
c. rhizomelia
d. horizontal acetabular roof
e. atlantoaxial instability
Atlanto-axial instability
Achondroplasia is the most commonly seen autosomal dominant rhizomelic dwarfism.
Rhizomelia refers to relative shortening of the proximal compared with the distal portion of
the limbs. Achondroplasia has widespread skeletal manifestations affecting the skull, chest,
spine, pelvis and extremities. Intelligence and motor function are normal. The most significant
complication is brain-stem or spinal cord compression due to spinal stenosis, which is caused
by alignment abnormalities and decreased spinal canal size due to short pedicles with a reduced
interpedicular distance. Atlantoaxial instability is defined as a predental space of 3 mm or more
in adults and 5 mm or more in children, or where there is considerable change between flexion
and extension. It is seen in inflammatory arthritides, in Down’s and Morquio’s syndromes, and
with retropharyngeal abscess in a child.
At which of the following skeletal locations does avascular osteonecrosis typically only occur
in the presence of an associated fracture? [B4 Q51]
a. medial tibial condyle
b. second metatarsal head
c. lunate
d. femoral head
e. proximal scaphoid pole
Proximal scaphoid pole
Osteonecrosis may be caused by two mechanisms: interruption of arterial supply, and intra- or
extra-osseous venous insufficiency. Interruption of vascular supply is usually associated with
a fracture, as seen in the proximal scaphoid following waist fractures. Femoral head
osteonecrosis can occur with subcapital fractures, or without fracture as in Legg–Calve´–
Perthes disease. Other common locations that may develop osteonecrosis without overt fracture
include the medial tibial condyle (Blount’s disease), metatarsal head (Freiberg’s infraction)
and the lunate (Kienbock’s disease). Radiographic findings often lag several months behind the injury or onset of symptoms, and MR is the most sensitive imaging modality. Radiographic
signs include focal radiolucencies, sclerosis, bone collapse and loss of joint space
Which of the following is not a recognized cause of myeloid hyperplasia (red marrow
reactivation/reconversion) in a 50-year-old adult? [B4 Q68]
a. sickle cell disease
b. smoking
c. chemotherapy
d. long-distance running
e. Gaucher’s disease
Gaucher’s disease
Marrow reconversion is the repopulation of yellow (fatty) marrow with haematopoietic cells, reconverting the fatty marrow to red marrow. This occurs when the haematopoietic capacity of the existing red marrow in an adult is insufficient. This can result from increased physiological requirement (long distance running), chronic anaemia (sickle cell disease) and chemotherapeutic treatment with granulocyte–macrophage colony-stimulating factor. The pattern of reconversion is predictable and it is the reverse of the age-related physiological conversion of red to yellow marrow. Reconversion begins in the axial skeleton and progresses distally through the appendicular skeleton, to end in the hands and feet. Knowledge and recognition of this pattern are important because neoplastic infiltration of adult yellow marrow in malignant disease may have similar MR appearances. However, malignant marrow replacement tends to have a more random distribution than reconversion; it may enhance with intravenous gadolinium, can cause cortical destruction and may extend into the soft tissues.
You are asked to image the pelvis with MRI for someone who has a hip arthroplasty. Which
of the following measures can be used to decrease the magnetic susceptibility artifact from the
joint prosthesis? [B1 Q33]
A. Use fast spin echo (FSE) imaging rather than GE imaging.
B. Choose a higher field strength magnet, e.g. 3 T rather than 1.5 T.
C. Position the long axis of the prosthesis perpendicular to the main magnetic field strength
(β0) direction if possible.
D. If fat saturation is to be employed, use spectral fat suppression rather than STIR imaging.
E. Increase the volume of the voxels (decrease spatial resolution).
Use fast spin echo (FSE) imaging rather than GE imaging.
Magnetization of the implant affects the local field gradient, proton dephasing, and spin
frequency, resulting in signal void, spatial distortion, and spurious high signal. The lack of a
180° rephasing pulse in GE sequences, as opposed to SE/FSE sequences, means that T2*
effects are not reversed, and a greater dephasing of spins occurs in GE than in SE/FSE
techniques. Thus, GE techniques have a greater sensitivity to magnetic susceptibility effects.
This is detrimental when imaging patients with metal prostheses, but can be used to advantage
in certain clinical situations, e.g. when identifying subtle haemorrhage due to the magnetic
susceptibility effects of iron in haemosiderin (a blood breakdown product)
A radiologist is reporting a 99m Tc bone scan and describes it as a ‘super-scan’. He can say this
because of reduced uptake in the: [B1 Q73]
A. brain
B. skeleton
C. kidneys
D. bowel
E. myocardium.
Kidneys
A super-scan refers to a99m Tc-labelled technetium IBS where there is diffuse increased osseous uptake with apparent reduced renal and soft tissue uptake. The appearance is commonly due to widespread osteoblastic bony metastases (e.g. prostate or breast carcinoma), but is also caused by non-malignant disease (e.g. renal osteodystrophy, hyperparathyroidism, osteomalacia, myelofibrosis, Paget’s disease). In metastatic disease there is usually higher uptake in the axial than the appendicular skeleton.
In IBS uptake is normally seen in bone, kidneys, and bladder, soft tissues (low levels), breasts (particularly in young women), and epiphyses (skeletally immature patients).
Uptake is seen in the myocardium (high), brain (high), and bowel (moderate) in FDG-PET scanning, not IBS; however myocardial uptake on IBS can be seen in cases of recent myocardial infarction and amyloidosis.
Note that poor renal function can often demonstrate reduced or absent renal visualization producing an appearance like a superscan (false positive), whereas urinary tract obstruction in prostatic carcinoma can increase renal activity and lead to false negative scans.
A 56-year-old woman who has had chronic wrist pain since a fall several months previously is
referred for an MR arthrogram of her wrist with a suspected triangular fibrocartilage complex
(TFCC) tear. Which of the following would be the best sequence for visualising a TFCC tear?
[B2 Q56]
a. T1 axial
b. T2 coronal
c. Gradient echo sagittal
d. T2 sagittal
e. T1 sagittal
T2 coronal
The best sequence would be a T2 or T2* image for detecting a tear. This is also a useful plane
in which to assess for ulnar variance; positive ulnar variance has an association with
perforations. The central portion of the articular disc is not well vascularised and therefore a
tear in this portion will heal poorly. The peripheral portion, however, has been vascularised.
You are reporting an MRI knee on a patient with moderately severe osteoarthritis (OA), as
diagnosed on plain film radiography. The patient describes significant knee pain. Which of the
following statements best describes the relationship between symptoms, plain film findings,
and MRI findings? [B1 Q27]
A. The MRI findings correlate well with the severity of findings on plain film radiography.
B. MRI findings correlate well with the patient’s symptoms.
C. Plain film findings correlate well with the patient’s symptoms, unlike MRI.
D. Plain film and MRI both correlates well with the severity of the patient’s symptoms.
E. Symptoms, plain film findings, and MRI findings do not have a significant association with
each other.
The MRI findings correlate well with the severity of findings on plain film radiography.
MRI has been shown to correlate well with the severity of OA as depicted on plain film
radiography. Neither MRI nor plain film appearances are significantly associated with the
patient’s symptoms.
You are reporting an MRI knee on a patient with moderately severe osteoarthritis (OA), as
diagnosed on plain film radiography. The patient describes significant knee pain. Which of the
following statements best describes the relationship between symptoms, plain film findings,
and MRI findings? [B1 Q27]
A. The MRI findings correlate well with the severity of findings on plain film radiography.
B. MRI findings correlate well with the patient’s symptoms.
C. Plain film findings correlate well with the patient’s symptoms, unlike MRI.
D. Plain film and MRI both correlates well with the severity of the patient’s symptoms.
E. Symptoms, plain film findings, and MRI findings do not have a significant association with
each other.
The MRI findings correlate well with the severity of findings on plain film radiography.
MRI has been shown to correlate well with the severity of OA as depicted on plain film
radiography. Neither MRI nor plain film appearances are significantly associated with the
patient’s symptoms.
You are discussing OA with a rheumatologist. He/she is curious to know what radiological
findings seen in early disease are associated with progressive, as opposed to stable, arthritis.
All of these are associated with OA, but which is least likely to indicate progressive disease?
A. Increased uptake on isotope bone scan.
B. Grade 1 osteophytosis in the knee.
C. Osteochondral defect.
D. A focal area of high signal on T2WI and STIR in the subchondral bone.
E. A serpiginous subchondral line that is low signal on T2WI and T1WI, with an adjacent
T2WI high signal line.
Grade 1 osteophytosis in the knee.
While the Kellegren and Lawrence grading of OA is the most widely used scale for grading
OA on plain films, early osteophyte formation is not definitively ‘arthritic’ change. Studies
have shown that patients with this type of early change infrequently progress to developing
more severe disease. Recent research in OA is focused on the impact of the subchondral bone
on the disease, rather than hyaline cartilage. This is evidenced by increased uptake on bone
scan being closely linked to progressive OA, even in patients with relatively normal joints on
plain film. The foci of high signal on T2WI and STIR are bone marrow lesions that if persistent
can indicate pathology in the subchondral bone, which can lead to arthritic change. The
serpiginous line describes the classical finding of avascular necrosis. Both this and
osteochondral defects lead to progressive OA change.
A 55-year-old female presents to the rheumatologists with a history of episodic swollen red
joints over the previous 2 years. She also complains of left hip pain. The patient’s rheumatoid
factor is not known at the time of requesting the radiographs. There is no other past medical
history. The rheumatologists have requested bilateral hand and pelvis x-rays. The hand x-rays show bilateral asymmetric disease affecting the distal and proximal interphalangeal (IP) joints.
In the affected distal IP joints there are central erosions, adjacent sclerosis, and marginal
osteophytes. The first carpometacarpal joint in the left hand shows loss of joint space with
osteophyte formation. The scaphoid-trapezium joint in the right hand also shows loss of joint
space and adjacent sclerosis. In the left hip there is nonuniform loss of joint space, with
associated subchondral cyst formation. What is the main differential? [B1 Q37]
A. RA.
B. Psoriatic arthritis.
C. Erosive OA.
D. Calcium pyrophosphate deposition (CPPD) arthropathy.
E. Ankylosing spondylitis
Erosive OA.
If the presence of erosions is ignored the pattern of disease is typical of OA. Erosive, or
inflammatory, OA shows typical OA distribution, but with central erosions in the affected joint
spaces in the distal IP, and less commonly proximal IP, joints. These central erosions in
combination with marginal osteophytes give the classical gull-wing appearance to affected
joints. Whilst individual joint appearance can be identical to psoriatic arthropathy, which can
precede the development of the skin disease, the overall pattern is atypical, making it less likely.
Also, while the distribution of CPPD arthropathy is often identical to OA, it would not cause
erosions, and would usually be associated with chondrocalcinosis.
An 80-year-old woman complains of pain in both hands. Radiography of the hands shows
bilateral central articular ‘seagull’ erosions affecting the interphalangeal joints of fingers in
both hands. Mild periarticular osteoporosis is seen. The most likely diagnosis is? [B5 Q18]
(a) Osteoarthritis
(b) Erosive osteoarthritis
(c) Calcium pyrophosphate deposition disease
(d) Gouty arthropathy
(e) Rheumatoid arthritis
Erosive osteoarthritis
Central articular erosion with a ‘seagull’ pattern, ankylosis and periarticular osteoporosis is
typical of erosive osteoarthritis. This is seen in older women and is usually limited to hands,
particularly affecting the proximal interphalangeal joints.
A patient presents to their GP with a complex history of acute episodes of severe tender
inflamed joints, around the knee. At present the patient has joint stiffness which is most
pronounced in the evenings and mild joint pain. The patient has a past medical history of
hypothyroidism. A plain film is requested which shows chondrocalcinosis and moderate
degenerative change in the lateral tibiofemoral compartment and the patellofemoral
compartment. Regarding CPPD disease, which of the following statements is the most
appropriate? [B1 Q47]
A. The presence of chondrocalcinosis indicates a radiological diagnosis of pseudogout.
B. Pseudogout syndrome is the most common means of presentation for this disease.
C. Disproportionate involvement of the patellofemoral joint is the most frequently seen
radiographic finding.
D. The presence of crystals displaying positive birefringence at polarized light microscopy
allows for the definitive diagnosis of pyrophosphate arthropathy.
E. The presence of hypothyroidism is associated with the diagnosis.
The presence of hypothyroidism is associated with the diagnosis.
Disorders associated with CPPD are the four Hs:
Hyperparathyroidism
Haemochromatosis
Hypothyroidism
Hypomagnesaemia.
Use of nomenclature in this disorder is confused.
Pseudogout is the clinical presentation of an acutely inflamed joint due to calcium pyrophosphate crystal deposition, and as such is not a radiological diagnosis. Pseudogout syndrome is the dominant feature in only 10–20% of cases of CPPD. Another 10–20% of cases are asymptomatic, whilst most present with symptoms identical to OA, with occasional flares.
Disproportionate involvement of the patellofemoral joint is a characteristic feature but is only occasionally seen. Pyrophosphate arthropathy is a description of the pattern of disease present due to crystal deposition, and as such is not a diagnosis made by analysis of joint aspirate.
A 55-year-old man presents with pain and swelling in the left big toe. The plain radiograph
shows periarticular erosions with sclerotic margins and overhanging edges in the first
metatarsophalangeal joint. The joint space is preserved and there is moderate surrounding soft
tissue swelling. The most likely diagnosis is? [B5 Q13]
(a) Rheumatoid arthritis
(b) Erosive osteoarthritis
(c) Gouty arthritis
(d) Psoriatic arthropathy
(e) Calcium pyrophosphate deposition disease.
Gouty arthritis
Periarticular erosions
sclerotic borders
overhanging margins
preserved articular surface
Rheumatoid arthropathy has non-proliferative marginal erosions, symmetrical distribution and joint space narrowing. Psoriasis show progressive joint destruction with erosions. Erosive osteoarthritis is symmetrical with erosions are in the centre of the articular surface.
Calcium pyrophosphate deposition disease is polyarticular, and shows chondrocalcinosis and joint-space narrowing.
A 28-year-old male presents with soft tissue swelling, pain, and reduction of motion in the
small joints of his hands. Plain films of the hands show erosions at the (MCP) joints and distal
interphalangeal joints with periosteal reaction and enthesophytes. What is the most likely
diagnosis? [B1 Q2]
A. Psoriatic arthropathy.
B. RA.
C. SLE.
D. Haemochromatosis.
E. Calcium pyrophosphate dihydrate crystal deposition disease.
Psoriatic arthropathy.
Bone involvement before skin changes is evident in up to 20% of cases. Nail pitting or
discolouration is common and correlated with the severity of the arthropathy. Five distinct
manifestations have been described: oligoarthritis, polyarthritis (predominately distal IP joints),
symmetric type (resembling RA), arthritis mutilans, and spondyloarthropathy. The
characteristic distribution involves the small joints of the hands and feet, with or without
spondyloarthropathy. Involvement in the hands tends to include distal IP as well as MCP or
PIP joints, with early tuft resorption and distal IP erosive disease. The erosions become so
severe that a ‘pencil-in-cup’ deformity and telescoping of the joint may occur. Bone density
can be normal, and the joint distribution is asymmetric. Similarly, sacroiliitis is asymmetric,
unlike ankylosing spondylitis and syndesmophytes, which are non-marginal and asymmetric;
in ankylosing spondylitis they are marginal and asymmetric. The spondyloarthropathy of
psoriatic arthropathy is indistinguishable from that of reactive arthritis, the clinical scenario
(rash vs uveitis/urethritis) providing the diagnosis.
The arthropathies of CPPD disease and haemochromatosis are essentially identical
radiographically. Chondrocalcinosis is commonly seen in the wrist (triangular fibrocartilage)
and knee (menisci). The joints most affected are the knee, wrist and second and third MCPs of
the hand: the IP joints tend to be spared. Early disease shows erosive change. More advanced
disease demonstrates sclerosis, osteochondral fragments, and osteophytes. Subchondral cysts
are common and large. RA is rarely found in the distal IP joints and periosteal reaction is not
a feature. SLE is usually non-erosive and affects the MCP joints.
An 18-year-old male patient presents to the rheumatologists with a history of proximal right
tibial pain and sternal pain. The patient has a history of psoriasis and is also being seen by the
dermatologists with palmoplantar pustulosis. Plain films of the sternum indicate sclerosis of
the manubrium and erosive disease in the sternoclavicular joint. Plain films of the tibia show a
lucent lesion in the proximal tibial metaphysis with associated periosteal reaction. An MRI
shows high signal on STIR in the proximal tibial metaphysis, with a cortical defect. This area
enhances on the T1 post gadolinium images, as does the periosteal region. A bone biopsy of
the region is negative except for inflammatory cells. What is the most likely diagnosis? [B1
Q22]
A. Psoriatic arthropathy.
B. Synovitis, acne, pustulosis, hyperostosis, and osteitis (SAPHO) syndrome.
C. Chronic recurrent multifocal osteomyelitis (CRMO)
D. Chronic osteomyelitis.
E. Aseptic necrosis.
SAPHO syndrome.
The most common presentation for this condition is acne and palmoplantar pustulosis with
changes in the sterno-costoclavicular region where hyperostosis is seen, often with some
erosive change. The condition can also cause several manifestations in the spine, ranging from
focal hyperostosis of one or more lumbar vertebrae, to a syndrome strikingly like psoriatic
spondylo-arthropathy. In the appendicular skeleton hyperostosis is commonly seen. SAPHO
can also give rise to manifestations identical to osteomyelitis, but with no causative organism.
In this respect there is overlap with CRMO syndrome. Patients are frequently human leucocyte
antigen (HLA) B27 positive, and psoriasis frequently coexists, leading some authors to suggest
that SAPHO is a variant of psoriatic arthropathy, although this is not widely accepted.
A 30-year-old man visits his general practitioner complaining of recent onset of acne and
discharging pustules on his palms. He has a history of several years of pain and swelling at the
medial end of his right clavicle. Radiographs of the shoulder demonstrate hyperostosis and
early ankylosis of the sternoclavicular joint. What is the most likely diagnosis? [B4 Q90]
a. SAPHO syndrome
b. suppurative osteomyelitis
c. psoriatic arthritis
d. Reiter’s disease
e. recurrent multifocal osteomyelitis
SAPHO syndrome
Synovitis, acne, pustulosis, hyperostosis and osteitis. (SAPHO) syndrome is a term encompassing several disease entities that demonstrate an association between rheumatological and cutaneous lesions. There may be a delay of several years between the onset of osseous symptoms and cutaneous manifestations. It is thought to be similar to chronic recurrent multifocal osteomyelitis in children. The dominant radiographic abnormality is new and bizarre bone proliferation, with the sternoclavicular joint affected in 70–90% of cases.
A 44-year-old female patient presents to the rheumatologists with a history of multiple painful
joints for 2 years. She has synovitis clinically, confirmed on ultrasound, which involves the
MCP joints bilaterally. PA and Norgaard views of the hands are requested and show small
erosions in the distal radio-ulnar joint and the piso-triquetral joint, but no erosions at the MCP
joints. There is widening of the scapholunate interval on the right side. There is ankylosis of
the capitate to the hamate on the left. There is periarticular osteoporosis. Which of these
features is atypical of RA?
A. Symmetrical disease.
B. Synovitis on ultrasound but no erosions radiographically.
C. Erosions noted in the radio-ulnar joint and radio-carpal joint preceding MCP erosions.
D. Bony ankylosis of the carpal bones.
E. Periarticular osteoporosis.
Bony ankylosis of the carpal bones.
Whilst fibrous ankylosis of the carpal and tarsal bones does occur, bony ankylosis is extremely
rare in RA. It is, however, common in JRA. There are a number of unusual findings, which if
present should indicate a diagnosis other than RA. Productive bone change (e.g. periostitis or
enthesopathy) is extremely unusual. Osteophytes are also uncommon in the absence of
advanced associated osteoarthritic change. The exception to this is the distal ulna, a feature
known as ulnar capping. The other features are all typical of RA.
You are carrying out an MRI on a patient with a known history of RA. The patient has minimal
erosions on plain film, but severe arthralgia. She is being considered for biologic therapy. The
clinicians have requested an MRI of her hands. This reveals symmetrical disease in both hands
with areas of high signal on T2WI and low signal on T1WI around the triangular fibrocartilage
complex (TFCC), the radio-carpal joint (RCJ), and the distal radio-ulnar joint (DRUJ). The
abnormal areas at the TFCC and RCJ enhance following administration of gadolinium, the
DRUJ does not. A delayed T1WI sequence displays uniform enhancement in all joints. What
do these findings indicate? [B1 Q12]
A. Hyper-vascular pannus at the TFCC and RUJ, with fibrous pannus at DRUJ.
B. Fibrous pannus at the TFCC and RUJ, with joint effusion at DRUJ.
C. Hyper-vascular pannus at the TFCC and RUJ, with joint effusion at DRUJ.
D. Fibrous pannus at the TFCC and RUJ showing differential enhancement.
E. Fibrous pannus at the TFCC and RUJ with hyper-vascular pannus at DRUJ.
Hyper-vascular pannus at the TFCC and RUJ, with joint effusion at DRUJ.
Hyper-vascular pannus is intermediate to high signal on T2WI and low signal on T1WI. It also
enhances, retaining enhancement on delayed imaging. Joint effusions can be difficult to
differentiate from hyper-vascular pannus on pre-contrast imaging. Following enhancement,
they show only delayed enhancement. Fibrous pannus is low signal on all sequences.
A patient who is HIV positive presents with knee and ankle pain and swelling. Clinical
examination is otherwise unremarkable. Initial radiographs reveal only a joint effusion. The
complaint resolves after 4 weeks. What is the most likely diagnosis? [B1 Q34]
A. Septic arthritis.
B. Psoriatric arthritis.
C. HIV-associated arthritis.
D. Acute symmetric polyarthritis.
E. Hypertrophic pulmonary osteoarthropathy (HPOA).
HIV-associated arthritis.
This is oligoarticular, asymmetric and peripheral. It primarily affects the knees and ankles. It
has a short duration of 1–6 weeks; radiography may reveal a joint effusion. Acute symmetric
polyarthritis also occurs in HIV. It behaves clinically like RA, but patients are negative for
rheumatoid factor. Features that help differentiate it from RA are periostitis and proliferative
new bone formation. Occasionally an erosive variety with little or no proliferative bone
formation occurs. Psoriatric arthritis has a higher prevalence among AIDS patients than in the
general population. HPOA is associated with P carinii pneumonia (PCP) in AIDS. Plain films
reveal periosteal reaction. Kaposi’s sarcoma uncommonly affects the bone but does so most commonly in Africa. Non-
Hodgkin lymphoma (NHL) is the second most common tumour in HIV infection. It can
produce lytic, sclerotic, or mixed lesions with a wide zone of transition; they are usually lytic.
Other musculoskeletal complications in AIDS include infections (cellulitis, osteomyelitis,
septic arthritis, pyomyositis, necrotising fasciitis), Reiter’s syndrome, undifferentiated
spondyloarthropathy, polymyositis, osteonecrosis (especially of the femoral head),
osteoporosis, rhabdomyolysis, and anaemia.
A 30-year-old man presents with backache and morning stiffness. Examination reveals loss of
spinal movement, uveitis, and upper zone end inspiratory fine crepitations on auscultation.
Which of the following statements is most correct in relation to the radiological features of the
underlying condition? [B1 Q39]
A. Romanus lesions (anterior or posterior spondylitis) are a late feature.
B. Syndesmophytes are better depicted on MRI than plain film.
C. Ankylosis involves the vertebral edges or centre.
D. Sacroiliac joint widening is not a feature.
E. Enthesitis appears as low signal within the ligaments on STIR imaging.
Ankylosis involves the vertebral edges or centre.
The question refers to ankylosing spondylitis. Ankylosis involves the vertebral edges or centre,
with bony extension through the disc. The former is thought to be secondary to a Romanus
lesion, the latter an Andersson lesion. Romanus lesions are irregularities and erosions involving
the anterior and posterior edges of the vertebral endplates and are the earliest changes of
spondylitis depicted on conventional radiographs. On MRI an Andersson lesion is depicted as
disc-related signal-intensity abnormalities of one or both vertebral halves of a discovertebral
unit. They are often hemispherically shaped. MRI is better than conventional radiography at
depicting Romanus lesions, Andersson lesions (spondylodiscitis), and most other abnormalities,
although ankylosis is equally well detected by both modalities.
Syndesmophtyes are difficult to detect on MRI. Plain radiography is superior in this respect
because of its superior spatial resolution; syndesmophtyes are seen as bony outgrowths of the
anterior vertebral edges. They occur in 15% of the vertebrae of patients. Apical pulmonary
fibrosis affects 1% of patients. Sacroiliac (SI) joint erosion and widening is an early feature,
and this may initially be more prominent on the iliac side of the joint, as the cartilage on that
side is normally thinner. Later in the disease, sclerosis and ankylosis occur and the SI joints
become symmetrically fused.
Enthesitis is most prominently seen when the interspinal ligaments, those that extend between
the spinous processes, and the supraspinal ligaments are affected. Ligamentous involvement is
characterized by an increased signal intensity on either STIR images or contrast-enhanced
T1WI fat saturated sequences. It may be associated with osteitis of adjacent bone marrow in
the spinous processes. Arthritis of the synovial joints (e.g. facet joints) and insufficiency
fractures (often spontaneous or after minor trauma) are also features of the seronegative
spondylarthritides.
A 24-year-old male patient is referred from the rheumatologists with a history of back pain and
hip pain. Plain films are carried out. These show bilateral sacroiliitis with erosive change on the iliac side on the left, but sacral and iliac erosions on the right. The imaging of the spine
reveals large non-marginal syndesmophytes in the thoracolumbar spine with a relatively
normal lower lumbar spine. The patient also complains of foot pain and plain films reveal
evidence of a retrocalcaneal bursitis with erosion of the calcaneus. Hand x-rays reveal small
erosions asymmetrically in the distal IP joints in both hands. What is the most likely diagnosis?
[B1 Q52]
A. Ankylosing spondylitis.
B. Reactive arthritis.
C. Psoriatic arthritis.
D. Erosive OA.
E. Adult Stills disease.
Psoriatic arthritis.
Ankylosing spondylitis causes a symmetrical sacroiliitis. The syndesmophytes associated with
this are marginal and fine. It also typically progresses superiorly from the lumbar spine. Both
reactive arthritis and psoriatic arthritis cause an asymmetric sacroiliitis and the
syndesmophytes are usually centred on the thoracolumbar spine and are non-marginal and
bulky. Retrocalcaneal bursitis and erosions, whilst more common in reactive arthritis, can
occur in psoriatic arthritis, and reactive arthritis would uncommonly affect the hands. Also,
with all other factors being equal, psoriatic arthritis is much more common than reactive
arthritis, even without the skin manifestations, which are absent in up to 20% at presentation.
In a 21-year-old man with symptoms of chronic back pain, pain in his feet, particularly the
great toe and metatarsophalangeal joints, and bilateral sacroiliitis on plain films, the most likely
diagnosis is: [B2 Q7]
a. Ankylosing spondylitis
b. Gout
c. Inflammatory bowel disease-related arthropathy
d. Reiter’s syndrome
e. Psoriatic arthritis
Reiter’s syndrome
Reiter’s syndrome is the association of urethritis, conjunctivitis and mucocutaneous lesions.
Sacroiliitis is usually bilateral but often persists asymmetrically. There is an association with
the HLA B27 antigen. Reiter’s has a predilection for the great toe and metatarsophalangeal
joints.
A routine pre-operative chest X-ray in a 62-year-old woman shows bilateral erosion of the
distal clavicles. Which one of the following conditions might be responsible? [B2 Q62]
a. Hypoparathyroidism
b. Rheumatoid arthritis
c. Langerhans’ cell histiocytosis
d. Ankylosing spondylitis
e. Sarcoidosis
Rheumatoid arthritis
Myeloma, hyperparathyroidism, metastases, cleidocranial dysplasia and Gorlin basal cell
nevus syndrome all cause absence of the outer end of the clavicle. Destruction of the medial
end of the clavicle is caused by metastases, infection, lymphoma, eosinophilic granuloma,
rheumatoid arthritis, and sarcoma.
A 15-year-old boy presents with pain and swelling in the hands. Radiographs show periarticular
osteopenia, loss of joint space at the metacarpophalangeal joints and widened bases in the
proximal phalanges. A periosteal reaction is seen in the metacarpal bones. What is the most
likely diagnosis? [B5 Q44]
(a) Juvenile rheumatoid arthritis
(b) Psoriatic arthropathy
(c) Scleroderma
(d) Systemic lupus erythematosus
(e) Dermatomyositis
Juvenile rheumatoid arthritis
The condition is usually seen in young people before the age of 16 years. In the hand, the
metacarpophalangeal and interphalangeal joints are usually affected. Chronic synovitis causes
enlargement of bones and epiphyses. Malalignment and subluxation are common. An
important feature of juvenile arthritis is periosteal reaction affecting the metacarpal and
phalangeal shafts.
A 57-year-old man with increasing pain and stiffness in his hands and feet and worsening back
pain presents to his GP. Plain films of his hands show sclerosis of the terminal phalanges and
several ‘pencil-in-cup’ erosions. There is destruction of the interphalangeal joint of his right
great toe with exuberant periosteal reaction. There is also erosion of the posterior margin of
the calcaneus. The most likely diagnosis is: [B2 Q40]
a. Reiter’s syndrome
b. Ankylosing spondylitis
c. Rheumatoid arthritis
d. Psoriatic arthritis
e. Osteoarthritis
Psoriatic arthritis
This is usually HLA B27 positive and is associated with skin and nail changes in most cases.
The hands are often described as having sausage digits, and erosions with ill-defined margins
are characteristic. Sacroiliitis is often present and most often bilateral. Within the axial skeleton,
there is often large bulky vertically orientated soft-tissue ossification giving a ‘floating’
osteophyte appearance.
Which of the following favours a diagnosis of rheumatoid arthritis rather than tuberculous
arthritis? [B3 Q32]
A. Periarticular osteopenia
B. Marginal erosion
C. Relatively late sparing of joint space
D. Joint effusion
E. Uneven and thick synovial proliferation
E
The remaining answers are also seen in TB arthritis. Even and thin synovium, large bone
erosions, rim enhancement around bone erosion and extra-articular cystic masses are more
frequently seen in TB arthritis
Ivory phalanx, tuft resorption, and pencil-in-cup deformity of the middle phalynx in a patient
with asymmetric arthritis are most likely to represent: [B3 Q49]
A. Psoriatic arthritis
B. Haemachromatosis
C. Rheumatoid arthritis
D. Reiter syndrome
E. Scleroderma
Psoriatic arthritis
Hands are more commonly affected in psoriatic arthropathy, whereas feet are more commonly
affected in Reiter’s
In a 50-year-old woman with arthralgia, which of the following favours rheumatoid arthritis
over Systemic Lupus Erythematosus (SLE)? [B3 Q48]
A. Prominent subluxation of metacarpal phalangeal joints (MCPJ)
B. Usually bilateral and symmetrical
C. Absence of erosion
D. Radiographically similar to Jaccoud’s arthropathy
E. Hyperextension of Distal Interphalyngeal (DIP) and flexion of Proximal Interphalyngeal
(PIP) joints
Hyperextension of DIP J and flexion of PIP J
A-D are features seen in SLE.
On plain radiographs of the hands in a middle-aged male patient complaining of bilateral joint
pain and swelling, which single feature is most likely to support a diagnosis of psoriatic arthritis
over rheumatoid arthritis? [B4 Q15]
a. new bone formation
b. joint space loss
c. periarticular osteoporosis
d. periarticular erosions
e. soft tissue swelling
New bone formation is the hallmark finding of psoriatic arthritis and is not seen in rheumatoid
arthritis. Conversely, periarticular osteoporosis is seen in rheumatoid but is not a feature of
psoriatic arthritis. Both conditions may cause soft tissue swelling (typically a sausage digit in
psoriatic arthritis), joint space loss and erosions, which are marginal in psoriatic and marginal
and/or central in rheumatoid arthritis. Another distinguishing factor is the distribution of
involved joints in the hands, which is typically, but not always, interphalangeal in psoriatic and
metacarpophalangeal in rheumatoid arthritis.
Which of the following features is not a recognized primary musculoskeletal manifestation of
the CREST (calcinosis, Raynaud’s phenomenon, oesophageal involvement, sclerodactyly and
telangiectasia) syndrome? [B4 Q66]
a. digital oedema
b. calcinosis
c. acro-osteolysis
d. osteoporosis
e. joint erosions
Osteoporosis
CREST syndrome represents the limited form of the autoimmune connective tissue disorder
scleroderma. Five-year survival rate is 50–67%. The two other types of sclerodermas are
generalized (also called systemic sclerosis) and localized (morphoea). Common findings are
digital soft-tissue oedema with sclerodactyly (tapered soft tissues), acro-osteolysis
(autoamputation) and calcinosis. There may be associated arthritis that shows erosions (also
seen in the ribs) or terminal phalanx resorption, with joint space narrowing a late sign.
Osteoporosis is not usually a feature except in the context of disuse.
A young man complains of early morning back pain and stiffness, and undergoes plain
radiographs followed by MRI of the whole spine. Which single feature is most likely to suggest
a diagnosis of psoriatic arthritis over ankylosing spondylitis? [B4 Q32]
a. syndesmophytes
b. para-syndesmophytes
c. asymmetrical sacroiliitis
d. ankylosis
e. patchy bone marrow oedema
Para-syndesmophytes
Seronegative spondyloarthritis is an umbrella term for inflammatory joint or spinal
conditions that are not associated with rheumatoid factor or rheumatoid nodules. There are five
described subgroups: ankylosing spondylitis, psoriatic arthritis, arthritis associated with
inflammatory bowel disease, reactive arthritis (e.g. Reiter’s syndrome) and an undifferentiated
subgroup.
The subgroups may overlap both clinically and radiologically, and the diagnosis is more easily
made based on clinical history and examination. Imaging plays a limited role in differentiation,
particularly early in the disease when there can be considerable overlap of appearances.
The main exception is the identification of para-syndesmophytes, which are seen almost
exclusively in psoriatic arthropathy. In addition, bone marrow oedema can involve the entire
vertebral body in psoriatic arthritis, which may be a further useful distinguishing feature.
Undifferentiated spondyloarthritis is diagnosed when there is no clinical or radiological
evidence of sacroiliitis. All types may eventually progress to ankylosis.
An elderly female patient has plain radiographs performed in an outpatient clinic for bilateral
painful, stiff hips, which demonstrate joint space narrowing. Which additional feature is more
likely to support a diagnosis of rheumatoid arthritis rather than osteoarthritis? [B4 Q58]
a. eccentric joint space loss
b. soft tissue swelling
c. subchondral sclerosis
d. subchondral cysts
e. protrusio acetabuli
Protrusio acetabuli
Even or eccentric joint space reduction representing cartilage loss is seen in both types of
arthritis and not a distinguishing diagnostic feature. Although osteoarthritis is said to be
classically eccentric, this is difficult to assess accurately on many hip radiographs, as they are
not routinely taken in the upright, weight-bearing position. Subchondral sclerosis and cysts are
typically associated with degenerative osteoarthritis. Although soft-tissue swelling is a feature
of rheumatoid arthritis, the depth of the hip joint and the copious surrounding soft tissues mean
that any synovial swelling is unlikely to be appreciated clinically or radiologically. Subtle
osteophytes (in osteoarthritis) or erosive change/osteoporosis (in rheumatoid arthritis) can
distinguish between the two entities. An inflammatory cause should be considered in young
adults with hip pain and, if protrusio or other abnormalities are found, the sacroiliac joints
should be examined
A 57-year-old female patient with a history of multiple myeloma is referred for imaging due
to a history of arthralgia primarily affecting the hands. The patient describes early morning
stiffness that eases through the day. The clinicians report a finding of synovitis clinically. Blood
results have revealed a raised ESR. Hand x-rays are carried out which reveal sharply defined
intra-articular marginal erosions at the MCP joints of the index and middle fingers bilaterally.
The joint spaces are well preserved. There are also well marginated subchondral cysts noted in
the carpal bones, again with joint space preservation. Soft tissue nodules are noted around the
wrist joints, which are not calcified. There is no evidence of juxta-articular osteopenia. No
osteophytes are noted. What diagnosis is most strongly suggested by these findings? [B1 Q64]
A. Gout.
B. CPPD.
C. RA.
D. Amyloidosis.
E. Wilson’s disease.
Amyloidosis.
There are a lot of conditions that are capable of mimicking RA. In these cases, a few key
features can help reach a diagnosis. The classic finding in gout is of non-marginal erosions, as
opposed to those described. Nevertheless, marginal erosions can occur with gout. An RA-type
picture in the presence of non-marginal erosions or calcified soft-tissue nodules (tophi) should
suggest this diagnosis. CPPD gives a more productive pattern of arthritis, such as seen with
OA, affecting the radio-carpal joint. Thus, it is often suspected when the appearance is of OA
with a ‘funny distribution’. Amyloidosis is suggested first by the history of MM. Involvement
of the hands is more commonly seen in amyloid secondary to prolonged dialysis but can be
seen when the amyloid is secondary to MM, when the wrists are often affected. Amyloid can
closely resemble RA in its distribution and the pattern of erosions. However, three important
features can help differentiate: amyloidosis classically preserves the joint space, is not usually
associated with periarticular osteopenia, and amyloidosis causes well-demarcated subchondral
cyst formation in excess to that expected from the degree of joint disease.
A patient under joint care of cardiology and nephrology has bulky soft tissue nodules, well-
marginated erosion, preserved joint spaces involving the wrists, elbows, shoulder and hip joints.
Which is the most likely diagnosis? [B3 Q50]
A. Multicentric reticulohistiocytosis
B. Amyloidosis
C. Ochranosis
D. Wilson’s disease
E. Haemachromatosis
B.
Multiple sites can be involved and there is characteristic preservation of joint spaces
Which is the most characteristic feature in haemochromatosis? [B3 Q19]
A. Chondrocalcinosis
B. Small subchondral cysts with a fine rim of sclerosis
C. Symmetric joint space narrowing
D. Generalised osteopenia
E. Hook-like osteophytes on the radial aspects of the metacarpal heads
Hook like osteophytes on the radial aspects of the metacarpal heads
Although all the answers are features, hook-like osteopohytes are the most characteristic.
Arthropathy is seen in 50%, and chrondrocalinosis in more than 60%, with knees being most
commonly affected.
A 70-year-old man has plain radiographs of the hands and knees for joint pain and swelling,
which show joint space narrowing and chondrocalcinosis. Which additional finding would
support a diagnosis of haemochromatosis over pseudogout? [B4 Q73]
a. periarticular calcium deposition
b. metacarpal hooked osteophytes
c. eccentric joint space narrowing
d. large subchondral cysts
e. intra-articular loose bodies
Metacarpal hooked osteophytes
Haemochromatosis is excess iron deposition in the tissues and can be either primary (autosomal
recessive genetic disorder) or secondary to ineffective erythropoiesis or iron overload. Skeletal
manifestations include osteoporosis (which is proportional to the extent of iron deposition),
small subchondral cysts, arthropathy (50%), concentric joint space narrowing,
chondrocalcinosis, and characteristic hooked osteophytes on the radial aspect of the metacarpal
heads. Other organs can be affected by iron deposition, most commonly the brain, liver,
pancreas and spleen
In imaging of focal bone lesions in the appendicular skeleton, which of the following
radiographic features is most likely to indicate an aggressive or malignant process? [B4 Q17]
a. cortical expansion
b. lytic process
c. periosteal reaction
d. multiple lesions
e. wide zone of transition
Wide zone of transition
The zone of transition relates to the interface between the tumour margin and the host bone. It
is an extremely important discriminator, particularly for lytic lesions. Lesions with a well-
defined margin (and therefore narrow zone of transition) are described as geographic and are
usually non-aggressive, whereas those with a wide zone of transition are termed ‘permeative’
and are often malignant or aggressive (such as in osteomyelitis). Cortical expansion without
destruction is seen in many benign or slow-growing conditions such as fibrous cortical defect
and aneurysmal bone cyst. Many bone lesions, both benign and aggressive, are lytic. Periosteal
reaction does not indicate an aggressive lesion as such, but the pattern of reaction can do so.
Multiplicity is not an indicator of malignancy, as it can be seen in benign and self-limiting
processes (such as multiple enchondromatosis and neurofibromatosis). Equally, a solitary
lesion may be malignant
Of the following types of periosteal reaction, select the one most likely to indicate a benign
process? [B4 Q24]
a. soap-bubble
b. sunray
c. hair-on-end
d. laminated
e. Codman’s triangle
Soap-bubble
Periosteal reactions are usually a radiographic manifestation of underlying bone disease. The
term ‘soap bubble’ refers to expansion of the cortex without destruction by a lytic bone lesion.
The intact cortex usually indicates a benign process, whereas cortical destruction is associated
with malignant or aggressive lesions. Sunray and hair-on-end reactions are spiculated forms of
periosteal reaction that occur following periosteal elevation by tumour, with tumour preventing
the subperiosteal space from filling with new bone. Laminated or ‘onion-skin’ reaction occurs
with both malignant and benign processes and indicates an intermittent or cyclical process.
Codman’s triangles are formed by elevation and then destruction of the periosteum. They are
usually related to malignant tumours but can also be formed by aggressive benign processes.
The presence of punctate, ring-like or arcuate calcification in a lytic bone lesion on plain
radiography is most commonly associated with which of the following matrix types? [B4 Q19]
a. osteoblastic
b. fibrous
c. cartilaginous
d. cellular
e. mixed
Cartilaginous
Chondroid tumour matrix may or may not calcify, but, if it does, the pattern is characteristically
in arcs or circles and is sometimes described as ‘popcorn’. Osteoid matrix when calcified is
usually dense and homogeneous like a cloud. Calcified fibrous matrix has a characteristic
ground-glass appearance, whereas a cellular tumour usually does not show matrix calcification.
A mixed matrix will show mixed characteristics.
A middle-aged man with no significant medical history undergoes a radiograph of the pelvis
for localized tenderness following a fall. Multiple longitudinally orientated, 2–10 mm rounded
densities similar to cortical bone are seen throughout the cancellous bone, in a diffuse
symmetrical pattern concentrated around the acetabulum. There is no fracture. What is the most
likely diagnosis? [B4 Q8]
a. osteopathia striata
b. osteopetrosis
c. bone metastases
d. melorrheostosis
e. osteopoikilosis
Osteopoikilosis
Osteopoikilosis is a rare condition causing multiple enostoses (bone islands), which are
asymptomatic and usually of no clinical significance. They represent deposits of normal
cortical bone within the cancellous bone. Osteopathia striata (Voorhoeve’s disease) is similar
to osteopoikilosis in appearance and is usually asymptomatic, but it consists of linear
longitudinal or sunburst striations rather than rounded densities. Osteopetrosis causes
generalized increase in bone density, whereas melorrheostosis is a cortical process giving a
‘flowing wax’ appearance, usually affecting only one side of the affected bone. While
metastases are plausible, the patient would probably be symptomatic and have evidence or a
history of a primary tumour
A 20-year-old man presents with an increasingly painful right thigh which is worse at night.
Plain films of the area show a lucent area measuring approximately 8–9mm in the distal femur
surrounded by extensive sclerosis. The most likely diagnosis is: [B2 Q4]
a. Osteoblastoma
b. Giant cell tumour
c. Brodie’s abscess
d. Osteoid osteoma
e. Chondroblastoma
Osteoid osteoma
This most commonly presents in the second and third decades. The male: female ratio is 2.5:1.
Classically it presents with increasing pain which is worse at night and often relieved with
aspirin. Spinal lesions often lead to painful scoliosis. Almost any site in the body may be
affected but the most common regions are the lower limb and spine
A 20-year-old athlete presented with chronic leg pain relieved with aspirin. The plain
radiograph shows 2–3 cm area of sclerosis and cortical thickening in the midshaft tibia. CT
shows a 1 cm lytic lesion with central mineralisation. MRI demonstrates a bone oedema pattern
but normal surrounding soft tissues. The most likely cause of underlying pathology is? [B5 Q9]
(a) Stress fracture
(b) Ewing’s sarcoma
(c) Osteomyelitis
(d) Enchondroma
(e) Osteoid osteoma
Osteoid osteoma
This typically presents as a small lytic lesion with central mineralisation surrounded by reactive
sclerotic area. On bone scans, the central nidus shows intense uptake surrounded by region of
lesser activity (double density sign). MRI demonstrates bone oedema pattern and typically
relieved with aspirin.
Stress fractures are transverse and linear. Ewing’s sarcoma and infection must be considered if
soft tissue involvement is seen on MRI.
An osteoid osteoma is thought to be the cause of painful scoliosis in a 20-year-old man. Which
of the following is the single best answer regarding osteoid osteomas? [B3 Q23]
A. The nidus appears sclerotic with surrounding lucency on CT
B. Most found in long bones of the lower limbs
C. The nidus does not enhance on CT
D. Reactive sclerosis around the nidus is uncommon
E. The nidus demonstrates decreased activity on bone scintigraphy
Most found in long bones of the lower limbs
Although osteoid osteomas can occur in any bone, they are most common in the
metadiaphyseal femur and tibia. The nidus appears lucent on radiographs, intensely active on
bone scan – with surrounding well-defined luceny (double doughnut sign), isointense to muscle
on T1 and variable SI on T2
A 33-year-old man presents with a 2-year history of a hard lump on the left middle f inger. A
radiograph shows a 2 cm, well-defined, round, densely sclerotic lesion attached to the cortex
of the proximal phalanx of the left middle finger. No cortical erosion or periosteal reaction is
seen. A bone scan shows no tracer uptake. The most likely diagnosis is? [B5 Q35]
(a) Enostosis
(b) Osteoma
(c) Parosteal osteosarcoma
(d) Osteochondroma
(e) Myositis ossificans
Osteoma
The best diagnostic clue is the densely sclerotic, well-defined lesion attached to the parent bone.
Latent lesions show no tracer uptake
A 19-year-old man presents to his general practitioner with a sudden onset of painful scoliosis.
His pain improves with prescribed aspirin while awaiting MRI. MRI reveals a localized area
of inflammatory change in the left pedicle of L1. Subsequent CT shows marked sclerosis in the same region with a 5 mm, cortically based central lucency. What is the most likely cause?
[B4 Q35]
a. plasma cell cytoma
b. osteosarcoma
c. osteoid osteoma
d. Brodie’s abscess
e. lymphoma
Osteoid osteoma
Osteoid osteoma accounts for 12% of benign neoplasms of bone. It is most located in the cortex
of long bones (50% in the femur and tibia) with 15% in the spine, typically the pedicle. It rarely
exceeds 15 mm in size. Young men are most affected, with pain as the predominant presenting
feature due to the extensive inflammatory reaction and vascularity of the lesion. With spinal
lesions this results in a painful positional scoliosis, though most patients experience
improvement of the pain with salicylates. The lucent central area or nidus represents the
underlying pathological process, with the surrounding sclerosis representing reactive
inflammatory change in normal bone. Treatment traditionally was surgical curettage, but
radiologically guided percutaneous radiofrequency ablation is now used.
The case of a 22-year-old male with typical clinical and radiographic features of osteoid
osteoma is discussed at the musculoskeletal multidisciplinary team meeting regarding
treatment planning. A decision is made to offer radiofrequency ablation (RFA). You have been
asked to consent the patient for the procedure. Which of the following statements is true? [B1
Q40]
A. Up to six repeat procedures may be required.
B. It is performed under local anaesthetic.
C. Biopsy is necessary to confirm diagnosis prior to treatment.
D. Complete symptom relief is seen in 90% after initial therapy.
E. RFA treatment of vertebral osteoid osteomas is contraindicated.
Complete symptom relief is seen in 90% after initial therapy.
Osteoid osteoma is a benign, but painful, bone tumour typically found in the lower limbs of
children and young adults. The use of CT-guided RFA is a safe and effective technique and is
the treatment of choice over open surgical approaches. Complete relief of symptoms is
observed in approximately 90% after initial therapy and is reported up to 100% for secondary
procedures. The procedure is performed under general or spinal anaesthesia. A typical clinical
history of night pain relieved by non-steroidal analgesia and radiographic features of the central
nidus are considered sufficiently diagnostic to proceed with RFA. The procedure should not be
performed if there are doubts regarding diagnosis. Osteomas within the spine are potentially
treated by RFA if the nidus is >1 cm from the dura/neural structures. Since most spinal
osteomas are located within the posterior elements, however, RFA is generally unsuitable.
A 25-year-old man presents with a 4-month history of increasing dull lower back ache. He is
otherwise systemically well. He has no neurological signs. An x-ray of the lumbar spine
demonstrates a slight scoliosis, with an enlarged sclerotic left pedicle of L3. A subsequent CTscan shows a 3-cm lucent focus within the left pedicle of L3, which has expanded the bone.
There is surrounding sclerosis. What is the most likely underlying diagnosis? [B1 Q5]
A. Osteoid osteoma.
B. Enostosis.
C. Osteoblastoma.
D. Osteomyelitis.
E. Intracortical haemangioma.
Osteoblastoma.
Osteoblastoma is similar both clinically and histologically to osteoid osteoma, but there are some differences that aid in distinguishing these two entities. Clinically osteoblastoma is typically less painful than osteoid osteoma and does not respond as well to Aspirin. An osteoblastoma in the neural arch of the spine is more likely to cause neurological signs, as these lesions are typically larger and more expansile than osteoid osteoma. The lucent nidus seen in osteoid osteoma is usually less than 1.5–2 cm in size, whereas the nidus in osteoblastoma is usually larger than 2 cm at diagnosis and has less surrounding sclerosis. The nidus may or may not have a calcific focus within, in both these diagnoses.
The appearance described in the question is the subgroup of osteoblastoma that has similar features to osteoid osteoma. Other appearances on imaging of osteoblastoma include an expansile lesion with multiple small calcifications and a peripheral sclerotic rim or, more rarely, an aggressive appearance with osseous expansion, bone destruction, infiltrating soft tissue, and intermixed matrix calcification.
An enostosis (or bone island) may be giant (greater than 2 cm) but should be well defined and densely sclerotic. It is possible a bone abscess could cause a lytic lesion with surrounding sclerosis, but the patient is systemically well, making infection less likely. A bone abscess is also unlikely to be as expansile as the lesion described. Intracortical haemangioma is a very rare diagnosis, usually within the cortex of a long bone, such as the tibia. On CT there is a hypoattenuating lesion, with spotty internal calcification or a ‘wire-netting’ appearance.
A 26-year-old man presents with dull pain in the left thigh not relieved with salicylates.
Radiograph shows a 3 cm expansile lytic lesion in the mid-shaft of the left femur, which shows
reactive sclerosis and periosteal reaction. The bone scan shows intense tracer uptake. The most
likely diagnosis is? [B5 Q34]
(a) Osteoid osteoma
(b) Osteoblastoma
(c) Osteosarcoma
(d) Osteomyelitis
(e) Aneurysmal bone cyst
Osteoblastoma
Also called giant osteoid osteoma when measuring > 2 cm.
Osteoid osteoma are < 2 cm, have a predilection for the axial skeleton and usually respond to
salicylates. Osteosarcomas show cortical destruction, mineralised matrix, and soft tissue mass.
Aneurysmal bone cysts show fluid–fluid levels and no matrix calcification.
In a 19-year-old male with painful scoliosis, a well-defined 3cm geographic osteolytic lesion
is seen in the right posterior seventh rib with slight expansion and sharp sclerotic margins. CT
is performed and shows punctate calcification within the lesion and adjacent sclerotic bone.
MRI shows low intermediate T1 and intermediate-high T2 signal with bone marrow oedema.
Which is the most likely diagnosis? [B3 Q39]
A. Giant cell tumour
B. Fibrous dysplasia
C. Enchondroma
D. Osteoblastoma
E. Aneurysmal bone cyst
Osteoblastomas
Osteoblastomas share clinical and histological features with osteoid osteomas and 3-12% occur
in ribs.
A 30-year-old woman undergoes plain radiographic imaging of the hand for a palpable, painful
hard lump on the dorsum. Plain radiographs show a well-defined bony mass applied closely to
the diaphysis of the second metacarpal. CT shows a wide-based pedunculated lesion with a
perpendicular orientation to the diaphysis, no cartilage cap, and a matrix of mature trabeculated
bone. What is the most likely diagnosis? [B4 Q54]
a. osteochondroma
b. multiple osteo-cartilaginous exostoses
c. bizarre paraosteal osteo-chondromatous proliferation
d. Codman’s tumour
e. dysplasia epiphysealis hemimelica
osteochondroma are the absence of angulation away from the nearby physis and a wide base
Bizarre paraosteal osteo-chondromatous proliferation
Bizarre paraosteal osteochondromatous proliferation (also known as Nora’s lesion) is a rare
condition usually seen in adults in the third and fourth decades of life. Osteochondroma-like
lesions are seen most at the proximal and middle phalanges, followed by the metacarpals and
metatarsals. A relationship to trauma has been suggested but not proven. Other locations that
may be affected include the long bones (especially those of the upper extremity), skull and jaw.
It is thought to be a similar process to that which gives rise to lesions in myositis ossificans,
reactive periostitis and subungual exostosis. On plain radiographs, a well-defined bony mass is
seen attached to the surface of the parent bone. Features differentiating this from
A 25-year-old man presents with a painful knee. A plain film reveals a lucent area with a wide
zone of transition in the distal femoral metaphysis. MRI reveals fluid–fluid levels. What is the
most likely diagnosis? [B1 Q29]
A. Aneurysmal bone cyst.
B. GCT.
C. Osteosarcoma.
D. Chondroblastoma.
E. Osteoblastoma.
Osteosarcoma
The telangiectatic variety of osteosarcoma does show fluid–fluid levels, as does malignant fibrous histiocytoma or any necrotic bone tumour. Telangiectatic osteosarcoma is highly vascular and contains necrotic tissue and blood, with tumour located only along the periphery and septa. MRI will thus reveal enhancing nodularity in the latter locations; this finding will be absent in the case of ABC or GCT. In addition to those mentioned in the stem, the plain film findings include bone expansion and cortical breakthrough.
Unlike the other lesions, osteoblastoma does not demonstrate fluid–fluid levels on MRI; it is more common in the posterior elements of the spine than in the long bones.
ABC, GCT, and chondroblastoma have a narrower zone of transition on plain film than telangiectatic osteosarcoma.
GCT is subarticular.
Chondroblastoma is epiphyseal
A 22-year-old man presents to his GP with pain in his right knee which is gradually worsening
in severity and is relatively resistant to analgesia. MRI of the knee demonstrates an area of
geographic bone destruction in the distal femur with a wide zone of transition. There is marked
aneurysmal dilatation of the bone and a fluid-fluid level is present within the lesion. The most
likely diagnosis is: [B2 Q11]
a. Plasmacytoma
b. Simple bone cyst
c. Giant cell tumour
d. Telangiectatic osteosarcoma
e. Parosteal osteosarcoma
Telangiectatic osteosarcoma
With the MRI finding described, the most likely explanation is that the lesion is a telangiectatic
osteosarcoma. This is a rare type of osteosarcoma with a mean age at presentation of 20 years.
The most common site is around the knee (62%). Fluid-fluid levels are also seen in giant cell
tumours and aneurysmal bone cysts.
Which of the following indicates telangiectatic osteosarcoma (TOS) rather than an aneurismal
bone cyst (ABC)? [B3 Q26]
A. Enhancing septa without nodularity on MR
B. Marked expansile remodelling of bone
C. Cortical thinning
D. Presence of osteoid matrix with septal regions on CT
E. Presence of haemorrhagic spaces
Presence of osteoid matrix with septal regions on CT
Thick peripheral septa with nodularity, presence of an osteoid matrix within nodular or septal
regions, and aggressive growth features such as cortical destruction indicate TOS rather than
ABC.
A 30-year-old man undergoes MRI of the whole of the left lower limb and pelvis for a mid-
femoral destructive lytic lesion identified on radiography that is thought to represent a primary
bone tumour. MRI shows that the disease is confined to the femur with a 5 cm diaphyseal lesion
and a 1 cm proximal metaphyseal skip lesion. No enlarged lymph nodes are identified. CT
scans of the chest, abdomen and pelvis show two metastatic nodules in the lower lobe of the
left lung. Subsequent biopsy confirms the diagnosis of osteosarcoma. The cancer is correctly
staged as which of the following? [B4 Q78]
a. T1 N0 M0
b. T1 N0 M1a
c. T2 N0 M1b
d. T3 N0 M1a
e. T3 N0 M1b
T3 N0 M1a
Complete staging of primary bone sarcomas is unusual in that it also incorporates the
histological staging once tissue diagnosis via biopsy or surgical resection is available. The local
TNM classification is T1 (single lesion less than 8 cm), T2 (single lesion over 8 cm) or T3 (skip
lesions of any size). Nodal staging is N0 (no nodes) or N1 (any number of metastatic nodes).
Metastatic spread is staged, accordingly, as M0 (no metastases), M1a (metastases to lung) or
M1b (any other distant site). Once histology is available, tumours can be staged I–IV
Of the following subtypes of osteosarcoma, which is associated with the most favourable 5-
year survival? [B4 Q53]
a. multicentric
b. periosteal
c. paraosteal
d. telangiectatic
e. soft tissue
Paraosteal
Osteosarcoma is the second most common primary malignancy of bone after multiple myeloma,
accounting for 15% of all primary bone tumours. It usually affects those aged 10–30. Ninety-
five per cent are of the primary osseous type and, of these, paraosteal osteosarcoma has the
most favourable 5-year survival rate of 80%. Other osteosarcomas of the primary osseous type
include periosteal (5-year survival rate 50%) and telangiectatic (less than 20%). Multicentric
refers to synchronous osteoblastic osteosarcomas at multiple sites. It occurs exclusively in
children aged 5–10 and carries an extremely poor prognosis. The soft-tissue type is rare,
representing only 1.2% of all soft-tissue tumours. These lesions are primary soft-tissue tumours
with no attachment to bone. Death occurs within 3 years in most cases, tumour size being the
major predictor of outcome
A 25-year-old woman attends A&E after falling onto her right hand. A plain film of her hand
is taken to exclude fracture. No bony injury is seen. On examination, however, there is painless
swelling of the right index finger which she says has been present for a few weeks. Incidental
note is made of a small central lesion within the medullary cavity of the middle phalanx of the
index finger. There is no cortical breakthrough or periosteal reaction but there is bulbous
expansion of the bone with thinning of the cortex. The lesion contains dystrophic calcifications.
This is most likely to represent: [B2 Q38]
a. Giant cell tumour of the tendon sheath
b. Unicameral bone cyst
c. Brown tumour
d. Enchondroma
e. Epidermal inclusion cyst
Enchondroma
This lesion is most likely to be an enchondroma. This is a benign cartilaginous growth in the
medullary cavity and is usually asymptomatic. It most commonly occurs in the small bones of
the hands and wrist but may also occur in the proximal humerus and proximal femur.
Epidermoid inclusion cysts are usually in the distal phalangeal tuft and there is often a history
of trauma. A bone cyst would be unusual in the phalanges.
A 20-year-old woman presents with pain after injury to the index finger. The radiograph shows
a 2 cm lytic lesion with a matrix containing calcifications. There is endosteal scalloping of the
cortex with cortical expansion, but no cortical breach. The most likely diagnosis is? [B5 Q33]
(a) Bone infarct
(b) Enchondroma
(c) Chondrosarcoma
(d) Juxtacortical chondroma
(e) Epidermoid cyst
Enchondroma
Radiographic features are typical and diagnostic. A bone infarct appears as a serpiginous area
with sclerotic margins and no endosteal scalloping.
Chondrosarcoma shows periosteal reaction and an associated soft tissue mass.
Plain radiographs of the hands in a young woman are performed for unilateral deformity. These
show multiple lytic lesions in the medullary cavities of the tubular bones with cortical
expansion and matrix mineralization, and associated Madelung deformity. The changes are
unilateral. What is the most likely diagnosis? [B4 Q23]
a. Maffucci’s syndrome
b. Ollier’s disease
c. Trevor’s disease
d. Lichtenstein–Jaffe´ disease
e. Morquio’s syndrome
Ollier’s disease
Ollier’s disease or multiple enchondromatosis is characterized by the presence of benign
intraosseous cartilaginous tumours. The estimated prevalence of the disease is 1 in 100 000.
The distribution and number of lesions are variable but are often unilateral and monomelic.
Complications include pain, skeletal deformities, limb length discrepancy (including
Madelung’s deformity) and the potential risk of malignant change to chondrosarcoma in 20–
50% of cases. The condition in which enchondromas are associated with haemangiomas is
known as Maffucci’s syndrome. Neither is usually inherited. Trevor’s disease is an epiphyseal
dysplasia, whereas Lichtenstein–Jaffe´disease is another name for fibrous dysplasia. Morquio’s
syndrome is one of the lysosomal storage disorders known as the mucopolysaccharidoses.
A 21-year-old man presents with multiple swellings and focal areas of bluish discoloration in
both hands. Plain radiograph of the hands show multiple, welldefined, expanded lytic lesions
in the metacarpals. These lesions show stippled calcifications in the matrix and cause cortical
thinning. Multiple small round calcifications are seen in the surrounding soft tissues. T he most
likely diagnosis is? [B5 Q17]
(a) Ollier’s disease
(b) Maffucci’s syndrome
(c) Metastases
(d) Diaphyseal aclasis
(e) Kaposi’s sarcoma
Maffucci’s syndrome
This diagnosis is a combination of multiple enchondromatosis and soft tissue haemangiomas.
Hand and foot involvement is common and severe. The soft tissue calcifications are phleboliths
from haemangiomas.
In Ollier’s disease, there is absence of haemangioma and phleboliths.
A 29-year-old woman presents with a painful right knee which has been worsening over the
previous few weeks. A plain film of the right knee shows an oval expansile lesion with a
radiolucent centre in the metaphyseal region of the proximal tibia. There is a sclerotic margin
and geographic bone destruction. There are internal septations and stippled calcification. There
is no periosteal reaction. The most likely diagnosis is: [B2 Q55]
a. non-ossifying fibroma
b. Chondroblastoma
c. Giant cell tumour
d. Chondromyxoid fibroma
e. Chondrosarcoma
Chondromyxoid fibroma
This is most seen in the second and third decades and the most common site is the long bones, most often the proximal tibia and distal femur. Non-ossifying fibroma is usually asymptomatic. The appearances of a chondroblastoma would be similar but this would most likely be epiphyseal in location and usually presents in a slightly younger age group.
A 16-year-old boy fell playing football and hurt his left knee. He has some difficulty weight-
bearing and presents to the A&E department. An x-ray of his left knee is performed. This
demonstrates a small joint effusion, but no fracture is seen. An approximately 3-cm diameter,
well-defined lucent bony lesion, with a thin sclerotic margin, is identified within the proximal
epiphysis of the tibia. No internal calcification is evident on plain x-ray. What is the most likely
diagnosis for this abnormality? [B1 Q23]
A. Chondromyxoid fibroma.
B. Enchondroma.
C. GCT.
D. Chondroblastoma.
E. Chondrosarcoma
Chondroblastoma.
These are radiolucent lesions that typically occupy the epiphysis of long bones in younger
people, usually before skeletal maturity. They tend to be less than 4 cm in size, with
approximately threequarters having a sclerotic border and one-third a calcified matrix seen on
plain radiographs.
Chondromyxoid fibromas are rare benign tumours occurring in predominantly the second and
third decades of life. They characteristically have sclerotic margins and appear lobulated or
‘bubbly’. They usually arise in the metaphysis of long bones with occasional diaphyseal
extension.
GCTs tend to occur in young adulthood following skeletal maturity. Patients usually present
with pain. The lesion is purely lytic, typically with well-defined, but non-sclerotic, margins.
When present in long bones, the lesions are typically metaphyseal, extending across a fused
epiphysis to a subarticular location. Periosteal reaction is atypical, but expansile remodelling,
cortical penetration, and soft-tissue extension may be seen.
An 18-year-old student who fell two stairs and landed on her left knee attends A&E
complaining of generalised knee pain but can weight bear. No acute bony injury is
demonstrated on plain film, however a pedunculated lesion arising from the femoral
metaphysis and extending away from the knee joint is seen. The lesion shows continuity with
both the marrow and the cortex. The most likely diagnosis is: [B2 Q28]
a. Osteochondroma
b. Osteoblastoma
c. Osteoid osteoma
d. Chondroblastoma
e. Chondromyxoid fibroma
Osteochondroma
The description is classic for an osteochondroma or osteocartilagenous exostosis. These lesions are the most common benign growths of the skeleton, are usually found incidentally and are usually asymptomatic unless complications arise. Complications include fracture, vascular compromise, bursa formation and malignant transformation into chondrosarcoma.
A 12-year-old boy presents with a hard lump around his right knee. A radiograph shows a bony
projection from the medial part of the tibial metaphysis with continuity of the cortex and
medulla of the tibia. What is the most likely diagnosis? [B5 Q32]
(a) Osteochondroma
(b) Parosteal osteosarcoma
(c) Chondrosarcoma
(d) Periosteal osteosarcoma
(e) Juxtacortical myositis ossificans
Osteochondroma
The best diagnostic feature of osteochondroma is continuity of the bony cortex and medulla
with the parent bone.
On MRI performed for a tender osteochondroma of the femoral metaphysis in an adult, which
feature is most useful in determining the presence of malignant change? [B4 Q42]
a. thickness of the cartilage cap
b. lesion size
c. compression of local nerves
d. fracture of the stalk
e. bursa formation
Thickness of the cartilage cap
Osteochondromas are the commonest bone tumours and are considered developmental
exostoses rather than true neoplasms. They represent 20–50% of benign and 10–15% of all
bone tumours. They are made up of cortical and medullary bone and an overlying cartilage cap.
The cortex and medulla of the osteochondroma are continuous with the underlying host bone.
They are typically orientated away from an adjacent distal joint. Lesions are frequently solitary,
but multiple lesions are seen in hereditary multiple exostoses, an autosomal dominant
syndrome. Malignant transformation occurs in 1% of solitary lesions and in 3–5% of patients
with hereditary multiple exostoses. After skeletal maturity, continued lesion growth,
particularly of the cartilage cap, is suggestive of malignant transformation. Although benign
lesions may reach 10 cm in size, the cartilage cap should not exceed 1.5 cm after skeletal
maturation. Any bone that develops by enchondral ossification may develop an
osteochondroma, the long bones of the lower extremity being most frequently affected.
A 20-year-old man is referred for suspected malignant transformation of an osteochondroma.
Which of the following is a cause for concern in an osteochondroma? [B3 Q17]
A. New lucency
B. Reduced scintigraphic activity
C. Growth before physeal closure
D. Asymptomatic nature
E. Cartilagenous cap > 0.5cm
New lucency
Concerning features for Osteochondroma malignant transformation include:
New lucency
Increased scintigraphic activity
Gowth after skeletal maturation
Pain after puberty
Cortical destruction
Cartilaginous cap > 1.5cm.
You are reviewing a plain film of pelvis of a 70-year-old woman with recent hip pain. She has
a past medical history of bronchial carcinoid. You notice thick, coarsened trabeculae of the left
iliac bone, but in comparison to a previous film there is an area of cortical destruction with
‘ring-and-arc’ calcification. There is no adjacent periosteal reaction. Which of the following is
the most significant pathology present? [B1 Q58]
A. Paget’s disease.
B. Chondrosarcoma.
C. Osteosarcoma.
D. Chondroblastoma.
E. Lung metastasis.
Chondrosarcoma
The findings describe development of chondrosarcoma in an area of Paget’s disease. While
osteosarcoma is more common than either malignant fibrous histiocytoma or chondrosarcoma
in Paget’s disease, the ‘ring-and-arc’ calcification in the vignette indicates chondroid rather
than osteoid calcification. Sarcomatous transformation in Paget’s is rare, occurring in
approximately 1% of cases, but should be suspected if there is new focal pain or swelling. Such
lesions, even osteosarcomas, are usually lucent. Periosteal reaction is often absent due to the
rapidity of bone destruction.
Other complications of Paget’s disease include those related to osseous weakening (deformity
and fracture), arthritis, neurological entrapment, and both benign and malignant GCT.
Chondroblastoma may have internal chondroid calcification (60%) but is a well-defined,
benign, lucent lesion with a sclerotic rim occurring in the epiphyses of children and young
adults. Bronchial carcinoid metastases are usually purely osteoblastic (i.e., sclerotic, not lucent).
Which of the following favours chondroma rather than chondrosarcoma? [B3 Q34]
A. Size > 3cm
B. Pain
C. Age > 30
D. Location in hands and feet
E. Permeation into soft tissues
Location in hands and feet
Chondromas are usually < 3cm, painless and in younger patients in the peripheral skeleton
Plain radiographs of the femur performed for pain reveal a centrally located lucent lesion in
the medulla with a partially calcified matrix. Which of the following features favours a
diagnosis of chondrosarcoma over enchondroma? [B4 Q100]
a. arc-and-ring matrix calcification
b. ground-glass matrix
c. multiple lesions
d. deep endosteal scalloping
e. lesion size over 5 cm
Deep endosteal scalloping
Distinction of enchondroma and intramedullary chondrosarcoma in the appendicular skeleton
proximal to the metacarpals/-tarsals is difficult radiologically. A series of 187 patients showed
that chondrosarcoma was associated with endosteal scalloping, with scalloping involving more
than two-thirds of the extent of the lesion being strongly suggestive of malignancy. Other
powerful discriminating factors identified as favouring chondrosarcoma were cortical
destruction, soft-tissue mass, periosteal reaction, radionuclide uptake at scintigraphy and pain
associated with the lesion. Chondrosarcoma also tended to be larger with a mean size of 10 cm
compared with 6.7 cm for enchondroma. A ground-glass matrix with arcuate calcification is
characteristic of both types of cartilaginous lesion. Multiple lesions may be seen in both
malignancy and enchondromatosis (Ollier’s disease).
A middle-aged woman, known to suffer from polyostotic fibrous dysplasia, presents with a
palpable, 3 cm, soft-tissue mass in the upper left thigh. MRI shows a relatively homogeneous,
smooth, well-defined lesion located in an atrophic quadriceps muscle, which returns low signal
on T1W images and high signal on T2W images. Following administration of intravenous
gadolinium, the lesion shows moderately intense heterogeneous enhancement. Whatis the most
likely pathological nature of the soft-tissue lesion? [B4 Q59]
a. soft tissue myxoma
b. malignant fibrous histiocytoma
c. soft-tissue cavernous haemangioma
d. multiple lipomatosis
e. rhabdomyosarcoma
Soft tissue myxoma
The association of fibrous dysplasia and soft tissue myxoma is well established and is
commonly termed Mazabraud’s syndrome. The key is identifying the relationship between the
bone and soft-tissue pathology, with the osseous features of fibrous dysplasia usually preceding
the formation of a soft-tissue mass. The condition is non-familial and more commonly affects
women, the thigh being the most common location. Typical MRI appearances are of a well-
defined lesion with signal intensity like water, and often a fat rind or adjacent muscle high
signal on T2W images is seen. Although uncommon, there have been reported cases of
malignant change into osteosarcoma
An 18year old man undergoes a TcMDP bone scan to investigate pain in the right hip. A ‘hot’
lesion is seen in the right proximal femur. No other lesions are seen. Which of the following
lesions would appear as ‘hot’ on a Tc MDP bone scan? [B2 Q53]
a. Osteopoikilosis
b. Fibrous cortical defect
c. Acute fracture within 12 hours of injury
d. Fibrous dysplasia
e. Haemangioma
Fibrous dysplasia
The most common site of monostotic fibrous dysplasia is the ribs, followed by proximal femur
and craniofacial bones. Three-quarters of cases present before age 30. Other benign lesions
causing a ‘hot’ on bone scan include Paget’s disease, brown tumours, aneurysmal bone cysts,
osteoid osteoma and chondroblastoma. Acute fractures are not usually ‘hot’ until after the first
24–48 hours.
A 70-year-old man undergoes CT of the skull for investigation of clinically apparent
macrocephaly confirmed on skull radiography. You are asked by the referring clinician to
review the images. Which finding is most likely to support a diagnosis of fibrous dysplasia
over Paget’s disease? [B4 Q91]
a. widened diploe
b. asymmetrical involvement of the skull
c. sparing of the paranasal sinuses
d. osteoporosis
e. ground-glass medulla
Ground-glass medulla
A ground-glass appearance is characteristic of fibrous dysplasia and is the most useful
discriminating factor. Other features of fibrous dysplasia of the skull that can help distinguish
it from Paget’s disease are symmetry of distribution, presence of a soft-tissue mass, cyst-like
changes, thickness of the cranial cortices, and involvement of the paranasal sinuses, maxilla,
sphenoid, orbits, and nasal cavity.
A 34-year-old man has an MRI of the lumbar spine for lower back pain. This is normal apart
from a focal lesion present in the L4 vertebral body. This is reported as a vertebral
haemangioma. Which of the following MRI characteristics does this lesion most likely have?
[B1 Q53]
A. decreased T1, decreased T2, decreased STIR
B. decreased T1, decreased T2, increased STIR
C. decreased T1, increased T2, decreased STIR
D. increased T1, decreased T2, decreased STIR
E. increased T1, decreased T2, increased STIR
F. increased T1, increased T2, decreased STIR
G. increased T1, increased T2, increased STIR
increased T1 increased T2 decreased STIR
The variable proportions of vascular and fatty soft-tissue elements influence the MRI
appearance of haemangiomas. Lesions with a predominantly fatty matrix show high signal
intensity on T1WI, intermediate to high signal intensity on T2WI, and loss of signal on STIR
or fat suppressed T2WI. If the vascular elements predominate, the lesions appear hypointense
on T1WI and extremely hyperintense on STIR and T2WI. If MRI is inconclusive, CT may be
helpful in identifying the typical pattern of haemangiomatous bone replacement, such as the
honeycomb, ‘soap bubble’ or ‘sunburst’ appearance.
A 40-year-old woman with lung cancer and multiple liver metastases presents with back pain
and left L5 radiculopathy. Plain radiography demonstrates prominent vertical striations in L4
vertebral body. MRI scans demonstrated a sequestrated disc pressing the left L5 nerve root in
the lateral recess. The L4 vertebral body shows a well-defined lesion, returning high signal on
T1 and T2. The lesion in the L4 vertebral body is? [B5 Q8]
(a) Intraosseous ganglion
(b) Metastatic deposit from lung cancer
(c) Haemangioma
(d) Osteoid osteoma
(e) Schmorl’s node
Haemangioma
Vertebral haemangiomas are visualised with fine or coarse vertical striations, commonly seen
in vertebral bodies. CT shows a dotted appearance in a fatty matrix. On MRI, they present as
lesions, returning high signal on T1 and T2 due to their high fat content.
Metastatic lesions are Intermediate signal on T1 and high on T2. Schmorl’s node affects the
end plates.
A middle-aged woman undergoes an MRI of the lumbar spine for longstanding lower back
pain. She has no specific neurological signs and is otherwise well. MRI shows some lower
lumbar spine facet joint arthropathy and a 22cm well-defined rounded lesion in the L3 vertebral body. This displays high signal on both the T1 and T2 sequences. The most likely explanation
for this lesion is: [B2 Q13]
a. Discitis
b. Lymphoma
c. Myeloma
d. Metastatic deposit
e. Haemangioma
Haemangioma
This is most likely to be a benign haemangioma. These are relatively common lesions seen as
incidental findings on spinal imaging. High signal on T1 imaging is indicative of the presence
of fat within the lesion. All the other conditions would give a low-signal lesion on T1 imaging.
Plain radiographs of the spine in a 40-year-old man performed following a roadtraffic collision
reveal a slightly expanded midthoracic vertebral body with coarse vertical trabeculations.
Subsequent CT shows a ‘polka-dot’ appearance to the same vertebral body in the axial plane.
What is the most likely disorder affecting the vertebra? [B4 Q31]
a. aneurysmal bone cyst
b. osteoid osteoma
c. haemangioma
d. compression fracture
e. osteopoikilosis
Haemangioma
Metastatic disease, myeloma and lymphoma are the most common malignant spinal tumours,
and haemangioma is the most common benign tumour of the spine. The appearances described
are characteristic of a vertebral haemangioma. On MRI, these lesions typically appear of
mottled low-to-high signal on T1W images depending on the degree of fat present, and of very
high signal on T2W images. Other primary osseous lesions of the spine are more unusual but
may exhibit characteristic imaging features that can help develop a differential diagnosis.
Radiological evaluation of a patient who presents with osseous vertebral lesions often includes
radiography, CT and MRI. The complex anatomy of the vertebrae means that CT is more useful
than conventional radiography for evaluating lesion location and assessing bone destruction.
The diagnosis of spinal tumours is based on patient age, topographic features of the tumour
and lesion pattern as seen on imaging.
A 35-year-male presents with pain in the thigh. A plain radiograph reveals an eccentric
expansile lucent lesion without a sclerotic margin but with a narrow zone of transition in the
distal femoral metaphysis and epiphysis, which extends to the joint surface. What is the most
likely diagnosis? [B1 Q6]
A. Osteosarcoma.
B. Giant cell tumour (GCT).
C. Metastasis.
D. Aneurysmal bone cyst.
E. Fibrous dysplasia.
Giant cell tumour (GCT).
This is the classical description and location of a GCT. They occur age 20–40 years, in the long
bones and occasionally the sacrum and pelvis. They are lucent, eccentric, and expansile but do
not usually produce sclerosis and produce a periosteal reaction in less than a third of patients.
They may have a multiloculated appearance. They originate in the metaphysis but extend to
the subchondral surface in the skeletally mature. MRI often reveals fluid–fluid levels and some
low signal on T2WI due to haemosiderin or collagen deposition. The major differential
diagnosis is an aneurysmal bone cyst (ABC), but this classically has a sclerotic margin and
usually occurs under 30 years of age (75% occur before the age of 20 years). Fibrous dysplasia
would usually present at a younger age in the metaphysis with extension into the diaphysis; a
trabeculated/ground-glass appearance is typical with a thick sclerotic margin and endosteal
scalloping. Metastasis would be relatively rare at this age and there is no mention of a primary
tumour. Osteosarcoma would have a more aggressive appearance with a wide zone of transition,
periosteal reaction, cortical destruction, and soft tissue extension.
A 26-year-old woman presents with a 2-year history of an enlarging soft tissue mass in her left
thumb adjacent to the interphalangeal joint. An x-ray of the left thumb shows a soft tissue
swelling with a large well-defined erosion seen affecting the distal metaphysis of the proximal
phalanx. There is no soft tissue calcification or evidence of arthropathy at the interphalangeal
joint. A subsequent MRI scan shows a 3.5-cm well defined soft-tissue mass, which is low signal
on T1WI and enhances post administration of gadolinium. The lesion is low signal on T2WI
and gradient echo (GE) imaging. What is the most likely diagnosis? [B1 Q9]
A. Ganglion cyst.
B. Peripheral nerve sheath tumour.
C. Lipoma.
D. GCT of the tendon sheath.
E. Soft tissue haemangioma.
GCT of the tendon sheath.
A GCT of the tendon sheath is a nodular form of PVNS. These tumours are intimately associated with a tendon sheath and are most located in the hand. They usually manifest as a small slow-growing mass, with or without pain. Radiographs may show no abnormality or non-aggressive remodelling of the adjacent bone. In MRI, these lesions are typically hypo- or isointense to muscle on T1WI and T2WI, owing to abundant collagen and haemosiderin, often with enhancement. This is like the findings of diffuse intra-articular PVNS, when the extent of haemosiderin deposition may cause hypointense nodules on T2WI and blooming artifact on gradient echo (GE) sequences. It must be stated that the degree of haemosiderin content may not always be enough to cause marked hypo-intensity on T2WI in GCT of the tendon sheath.
A ganglion cyst could occur in this location and be related to a tendon sheath, but on MRI it is typically hyperintense on T2WI secondary to its fluid component. There may be thin rim enhancement of the wall post administration of gadolinium. Peripheral nerve sheath tumours are typically hyperintense on T2WI with variable contrast enhancement. Lipomas are similar in signal characteristic to subcutaneous fat on MRI, i.e. hyperintense on both T1WI and T2WI.
A soft-tissue haemangioma may contain phleboliths on plain radiographic imaging. On MRI, haemangiomas may be well circumscribed or have poorly defined margins, with varying amounts of increased T1WI signal owing to either reactive fat overgrowth or haemorrhage. Areas of slow flow are typically hyperintense on T2WI, while rapid flow can demonstrate a signal void on images obtained with a non-flow-sensitive sequence.
A 34-year-old woman presents with pain and swelling of the right knee over the previous 2
months. Plain films demonstrate a well circumscribed, expansile, lytic lesion eccentrically
located in the subarticular region of the right distal femur. The lesion has a narrow, non-
sclerotic zone of transition. What is the most likely diagnosis? [B4 Q1]
a. giant cell tumour
b. enchondroma
c. fibrous cortical defect
d. fibrous dysplasia
e. aneurysmal bone cyst
Giant cell tumour
Most giant cell tumours occur in patients with closed epiphyses, and although they may
originate in the metaphysis, lesions typically involve the epiphysis and abut the subarticular
surface. They are classically eccentrically located lesions with a narrow zone of transition, no
sclerosis, and no internal matrix mineralization. Giant cell tumours tend to be locally
aggressive, with a high recurrence rate after initial treatment. Enchondromas are the
commonest benign cystic lesion of the phalanges, though they are also seen in the long bones.
However, those in the long bones almost always contain calcified chondroid matrix.
Aneurysmal bone cysts are often seen as an eccentric lytic expansile lesion, but patients are
nearly all under the age of 30. Monostotic fibrous dysplasia is more commonly seen in the
proximal femur than distally, and lesions tend to have a sclerotic margin. Fibrous cortical
defects are asymptomatic lesions seen in children, which usually regress spontaneously, so they
are only rarely seen after the age of 30. They typically appear as lytic lesions with a thin
sclerotic border in the metaphysis of a long bone.
Considering the imaging features of extraarticular Pigmented Villonodular Synovitis (PVNS):
[B3 Q29]
A. Typically manifests as a soft tissue mass in 20% of cases
B. Osseous abnormalities are present in the vast majority
C. Extrinsic erosion is the most common osseous abnormality
D. Radiographs are normal in 80% of cases
E. Extensive erosions are more common in the knee
Extrinsic erosion is the most common osseous abnormality
Manifests as a soft tissue mass in 50-70% of cases, with normal radiographs in 20% and
osseous abnormalities in 5-25%. The most common osseous abnormality are extrinsic erosions
which are more often present in the ankle and foot.
Which is the most common location for giant cell tumour? [B3 Q18]
A. Proximal tibia
B. Proximal femur
C. Distal femur
D. Proximal fibula
E. Patella
Distal femur
50-65% occur around the knee but are rare in the patella
A 40-year-old mother presents with knee pain. Plain radiographs show an eccentric, expansile,
lytic lesion with a narrow zone of transition in the lateral femoral condyle extending to
subarticular bone. The lateral cortex is thinned but no periosteal reaction or sclerosis is seen.
MRI shows a well-defined bony lesion with intermediate signal on T1 and mixed signal on T2
and multiple fluid–fluid levels. The most likely diagnosis is? [B5 Q10]
(a) Benign fibrous histiocytoma
(b) Giant cell tumour
(c) Telangiectatic osteosarcoma
(d) Brodie’s abscess
(e) Simple bone cyst
Giant cell tumour
Typical features on radiograph are a lytic, eccentric, expansile, subarticular lesion with ‘soap
bubble’ appearance (fluid–fluid level on MRI).
Benign fibrous histiocytoma and telangiectatic osteosarcoma can also show fluidfluid levels
but are metaphyseal lesions not extending to the subarticular surface. A simple bone cyst is
unilocular and situated away from the articular surface.
A 15-year-old boy attended the Accident & Emergency Department with ankle pain after a
twisting injury 7 days previously. The history suggests there has been ill-defined swelling and
ache for a few weeks. The plain radiograph shows a fracture in the distal fibula, with lamellar
periosteal reaction. There appears to be an associated soft tissue bulge. What is the most likely
diagnosis? [B5 Q5]
(a) Fracture with large haematoma
(b) Neuroblastoma metastasis
(c) Lymphoma
(d) Ewing’s sarcoma with fracture
(e) Osteomyelitis
Ewing’s sarcoma with fracture
For a simple fracture with haematoma, this case has presented too early for a periosteal reaction.
Neuroblastoma metastasis could be considered in a child less than 5 years old, and lymphoma
should be considered in patients over 30 years age. Osteomyelitis could have been possible if
there had been a previous history of localised pain, fever etc.
Which of the following causes parallel spiculated (hair-on end) rather than divergent speculated
(sunray) periosteal reaction? [B3 Q7]
A. Osteosarcoma
B. Ewing’s sarcoma
C. Sigmoid colon cancer metastasis
D. Hemangioma
E. Meningioma
Ewing’s sarcoma
Other causes include syphilis and infantile cortical hyperostosis.
Which of the following is a cause of a mixed sclerotic/lytic lesion with a button sequestrum?
[B3 Q12]
A. TB
B. Ewing’s sarcoma
C. Eosinophilic granuloma
D. Metastases
E. Osteosarcoma
Eosinophilic granuloma
Eosinophilic granuloma is a cause of a mixed sclerotic/lytic lesion with a button sequestrum
Which of the following demonstrates the most uptake on PET/CT? [B3 Q44]
A. Primary bone lymphoma
B. Osteosarcoma
C. Chondrosarcoma
D. Enchondroma
E. Osteochondroma
A Greatest FDG uptake occurs in primary bone lymphoma and Ewing’s sarcoma. Osteosarcoma demonstrates moderate uptake. Most benign bone lesions are non-FDG avid, except for high giant cell containing tumours (Giant Cell Tumors (GCT), osteoblastomas, aneurysmal bone cysts) and fibrous lesions (fibrous dysplasia)
A ‘fallen fragment’ seen within a lytic bone lesion is most commonly associated with which
of the following? [B4 Q13]
a. aneurysmal bone cyst
b. unicameral (simple) bone cyst
c. giant cell tumour
d. eosinophilic granuloma
e. chondroblastoma
Unicameral (simple) bone cyst
The fallen fragment is virtually pathognomonic for a simple bone cyst. It represents a fragment
from a pathological fracture through the lesion, which has fallen to lie in a dependent location
in the cyst matrix
A 10-year-old boy presented with fracture of the left proximal humerus sustained during a
tackle in a football match. Plain radiographs show a pathological fracture and underlying lytic
lesion in the metaphysis of the proximal humerus. The lesion shows endosteal scalloping and
a small bone fragment in the floor of the cyst. MRI features include intermediate signal on T1
and high signal on T2 with a fluid–fluid level. What is the most likely diagnosis of the
underlying bony lesion? [B5 Q4]
(a) Unicameral bone cyst
(b) Lymphoma
(c) Aneurysmal bone cyst
(d) Telangiectatic osteosarcoma
(e) Giant cell tumour
Unicameral bone cyst
This is usually asymptomatic unless fractured. The lesion usually is 2–3 cm in size, usually in
the metaphysis, with its long axis parallel to the long axis of the bone. There may be endosteal
scalloping of the bone and a ‘fallen-fragment sign’ (if fractured, centrally dislodged fracture
fragment falls into a dependent position in the cyst).
A 16-year-old boy presents with a slowly enlarging, painful swelling in his left lateral chest
wall. A CXR shows an expansile lucent lesion arising from the lateral aspect of the left seventh
rib. An MRI scan is performed for further evaluation, and this demonstrates a lobulated, thin-
walled multiseptated lesion with fluid–fluid levels, the dependent layer of which are
hyperintense on T1WI. What is the most likely diagnosis? [B1 Q43]
A. Fibrous dysplasia.
B. Aneurysmal bone cyst.
C. Enchondroma.
D. Chondroblastoma.
E. Cystic angiomatosis.
Aneurysmal bone cyst.
ABC accounts for approximately 5% of primary rib lesions, excluding myeloma. The
radiological findings and age described in the question are classical for this lesion.
Approximately 75% of patients are <20 years of age. The key findings on MRI are the fluid–
fluid levels due to the settling of degraded blood products within the cysts. Fluid–fluid levels
may also be a feature of other lesions, including GCT and chondroblastoma, but the thin, well-
defined margins of an ABC should help to distinguish it from other lesions, particularly at this
young age group.
Fibrous dysplasia is the most common benign rib lesion. The radiographic appearances are
variable but may show unilateral fusiform enlargement and deformity with cortical thickening
and increased trabeculation of one or more ribs. The matrix may appear lytic, may demonstrate
a ground-glass appearance, or rarely be sclerotic. Amorphous or irregular calcifications may
be seen within the lesion on CT, and MRI shows low to intermediate signal on T1WI and
variable T2WI signal.
Enchondromas more typically arise in the anterior cartilaginous portion of the rib. Radiographs
reveal a lobulated, well-demarcated osteolytic lesion that demonstrates mild expansion and
well-defined, sclerotic margins. There is typically matrix calcification and CT is more sensitive
at detecting this when the calcification is subtle. MRI shows T2WI hyperintense foci that
appear to coalesce with one another and reflect the high fluid content of hyaline cartilage.
Chondroblastoma of a rib is reported in the literature but would be exceedingly rare and
typically occurs in the epiphyses of long bones. Cystic haemangiomatosis is a rare disease of
disseminated multifocal haemangiomatous or lymphangiomatous lesions in the skeleton and is
usually an incidental asymptomatic finding.
A 17-year-old girl presents with pain in the distal forearm which has worsened over the last six
to eight weeks. Plain films show an eccentric lytic radiolucency in the distal radius with a soap-
bubble appearance. The most likely pathology is: [B2 Q1]
a. Enchondroma
b. Aneurysmal bone cyst
c. Simple bone cyst
d. Fibrous dysplasia
e. Chondroblastoma
Aneurysmal bone cyst
Aneurysmal bone cyst is most common in females and 75% occur under 20 years of age. The
classic presentation is of pain of relatively acute onset with a rapid increase in severity over 6–
12 weeks. Common locations include the spine, with a slight preponderance for the posterior
elements, and the metaphysis of long bones – femur, tibia, humerus, and fibula. The lesion is
usually expansile with thin internal trabeculations giving it the characteristic soap-bubble
appearance.
A 35-year-old man sprains his right ankle and attends the A&E department. An x-ray of the
right ankle is performed. This does not show any evidence of a fracture, but the lateral view
does demonstrate a well-defined radiolucent lesion with a faint sclerotic margin in the mid calcaneus. There is some central calcification within the lesion. What is the most likely
diagnosis? [B1 Q28]
A. Simple bone cyst.
B. Normal variant.
C. Enchondroma.
D. Intraosseus lipoma.
E. Bone infarct.
Intraosseous lipoma.
This is usually asymptomatic and discovered as an incidental finding in adults between 30 and
60 years. The calcaneus is the most common site for intraosseous lipoma, accounting for
approximately 32% of cases. The key radiographic features are as described. The central
dystrophic calcification seen in approximately 62% of cases is considered pathognomic.
The major differential diagnosis of this lesion is a simple bone cyst, although this would not
contain central calcification. Unlike bone cysts elsewhere, it is seen at this site into adulthood.
Also within the differential diagnosis is a pseudo lesion within the calcaneus, caused by a
relative paucity of trabecular bone at the same location. Again, central calcification would not
be a feature in this phenomenon.
Enchondroma is typically a well-defined osteolytic lesion with central calcification, but it
usually has a predilection for tubular bones and would be exceedingly rare in the calcaneus.
The described appearances are not typical for bone infarct.
A 53-year-old woman attends A&E with a short history of dull right heel pain. She is otherwise
fit and well and there is no history of trauma. Plain radiographs of the right foot and ankle
reveal a 2cm expansile non-aggressive lesion in the calcaneum. It has a thin, well-defined
sclerotic border. There is no periosteal reaction but there is a small calcified central nidus. The
most likely cause of the lesion is: [B2 Q58]
a. Aneurysmal bone cyst
b. Intra-osseous lipoma
c. Lipoblastoma
d. Fibrous dysplasia
e. Desmoplastic fibroma
Intra-osseous lipoma
The calcaneum is a common location for an intra-osseous lipoma. They do, however, also occur
in the extremities, skull and mandible. There is no periosteal reaction unless there is an
associated fracture. Imaging features would be like those of a unicameral bone cyst. They are
often asymptomatic but can present with localised bone pain.
A 34-year-old sedentary male office worker presents with a 2-month history of heel pain. A
radiograph demonstrates a well-defined lytic lesion in the calcaneum. T his produces mild
expansion with endosteal scalloping and has a central ossified nodule. On MRI, the lesion is
high signal on T1 and T2. What is the most likely diagnosis? [B5 Q3]
(a) Giant cell tumour
(b) Fibrous cortical defect
(c) Intraosseous lipoma
(d) Osteoid osteoma
(e) Solitary bone cyst
Intraosseous lipoma
This is an expansile, non-aggressive lesion usually seen in metaphyses. It may contain a focal
area of dystrophic calcification within (secondary to fat necrosis). On MRI, the lesion returns
fat signal on all sequences (high signal on T1 and T2, and low signal on STIR).
A 50-year-old man who has been previously well presents with low back pain. Plain film
reveals an osteolytic midline lesion in the lower sacrum containing secondary bone sclerosis
in the periphery, as well as amorphous peripheral calcifications. A lateral film shows anterior
displacement of the bladder and rectum. He subsequently develops faecal incontinence. No
additional lesions were discovered after imaging of the whole spine. What is the most likely
diagnosis? [B1 Q1]
A. Osteomyelitis.
B. Ewing’s sarcoma.
C. Chordoma.
D. Myeloma.
E. Sacrococcygeal teratoma.
Chordoma.
Plain film is very insensitive for detecting sacral lesions. Metastatic disease is much more
common in the sacrum than primary malignancy. Chordoma is the most common primary
sacral lesion.
It is derived from the embryonic remnants of the notochord and is thus almost always found in
the midline or a paramedian location with respect to the spine. It is most found in the sacrum
(50%), clivus (35%), and vertebrae (15%). A chordoma manifests as a destructive, lytic lesion,
commonly with internal calcifications, at both plain radiography and CT. A large presacral
soft-tissue component is usually present, as are soft-tissue components within the sacrum and
sacral canal. Symptoms can include pain, sciatica, and rectal bleeding as well as other bowel
and bladder symptoms, reflecting compromise of sacral nerves. The tumours can extend across
the adjacent disc space and sacroiliac joint.
On MRI, chordomas demonstrate low to intermediate signal intensity on T1WI and prominent
increased signal intensity on T2WI. Enhancement of the soft-tissue components is variable, yet
often moderate, on both CT and MR images. Chordomas demonstrate a prominent vascular
stain at angiography. They are locally aggressive and develop in locations that do not permit
easy surgical cure. There is an almost 100% recurrence rate; tumour seeding along biopsy tracts
and surgical incisions can lead to multicentric local recurrences. Metastasis occurs in 5–43%
to liver, lung, regional lymph nodes, peritoneum, skin, and heart. The 5-year survival rate is
66% for adults.
Osteomyelitis in the sacrum is most often due to contiguous spread from a suppurative focus
and we are told this patient was previously well. Ewing sarcoma would occur at a younger age
(peaking at 15 years, 90% manifest between the ages of 5 and 30). In the case of myeloma it
would be atypical for the rest of the spine to be uninvolved. The sacrococcygeal region is the
most common location of teratomas discovered in infancy. It is only rarely discovered in
adulthood. Teratomas are composed of a mixture of cystic and solid components.
The other lesions to be included in the differential diagnosis of such a sacral mass are metastasis,
sarcomas, GCT, chondrosarcoma, and ependymoma.
A 65-year-old man undergoes radiographs of the lumbar spine and pelvis for lower back pain.
A destructive lytic lesion is identified in the midline of the inferior sacrum with internal areas
of calcification. Subsequent MRI reveals a heterogeneous lesion replacing much of the sacrum,
which returns moderate low signal on T1W and high signal on T2W images, with a soft-tissue
component extending into the presacral soft tissues. The lesion shows patchy moderate
enhancement with intravenous gadolinium. What is the most likely diagnosis? [B4 Q94]
a. metastasis
b. giant cell tumour
c. aneurysmal bone cyst
d. chordoma
e. plasmacytoma
Chordoma
Chordomas arise from notochordal rests and therefore almost always occur in the midline. They
are the most common primary malignant sacral tumour and account for 2–4% of all malignant
tumours of bone. They are found at all ages but most commonly occur in the fourth to seventh
decades of life. Approximately half develop in the sacrococcygeal region. There is usually a
large soft-tissue component, and the tumour may extend across the intervertebral disc space or
sacroiliac joint. Overall, the most common sacral lesion is metastasis due to the high red
marrow content, but other primary malignant lesions include myeloma, Ewing’s sarcoma and
lymphoma. The most found benign tumours are giant cell tumours and aneurysmal bone cysts.
Despite being relatively common in the rest of the spine, haemangiomas and osteoid osteomas
are rare.
Plain film, CT and MRI are performed for the investigation of suspected chordoma. Which is
the best answer? [B3 Q24]
A. Radiographic appearances show sacral osteosclerosis
B. Coarse calcification often present with associated soft tissue
C. Areas of low attenuation within a mass on CT
D. Intermediate SI on T2
E. Arise from the spinal canal
Areas of low attenuation within a mass on CT
Chordoma usually appears as a low attenuation mass on CT
A 60-year-old man with several months’ history of back pain, worse when sitting, and with no
bowel or bladder symptoms, undergoes evaluation with MRI. This shows a lobulated presacral
mass, low SI on T1 with several areas of high SI within it, most likely to represent areas of
calcification and haemorrhage. Which is the most probable diagnosis? [B3 Q25]
A. Chordoma
B. Chondrosarcoma
C. Myxopapillary ependymoma
D. Metastasis
E. Giant cell tumour
Chordoma
This is the most common primary malignant tumour of the sacrum. Giant cell tumours are the
second most common cause and are indistinguishable from chordomas on MRI.
A 24-year-old woman presents with worsening frontal headaches and a sixth nerve palsy. A
non-enhanced CT shows a lesion situated within the clivus with associated bony destruction;
there is soft-tissue extension into the nasopharynx. MRI shows a large inter-osseous mass
which is isointense to brain T1-weighted imaging and hyperintense on T2. The most likely
diagnosis is: [B2 Q47]
a. Sphenoid sinus cyst
b. Meningioma
c. Nasopharyngeal carcinoma
d. Metastasis
e. Spheno-occipital chordoma
Spheno-occipital chordoma
The most likely cause is a spheno-occipital chordoma. This is associated with bony destruction
in 90% of cases and is most usually within the clivus. Other sites include the sella, petrous
temporal bone, floor of middle cranial fossa and jugular fossa. Sacrococcygeal chordoma is the
most common subtype of chordoma and is usually located with the fourth or fifth sacral
segments. Vertebral/spinal chordoma accounts for only 15–20% of all chordomas and is most
often situated in the cervical spine.
A 60-year-old woman undergoing follow-up CT under the care of the oncologist develops a
new expansile lytic lesion. Which of the following primary tumours usually causes an expansile
lytic metastasis? [B3 Q6]
A. Cervix
B. Uterus
C. Ovary
D. Thyroid
E. Rectum
Thyroid
Renal cell carcinoma also causes expansile lytic metastases
lytic lesions within the scapula and clavicle secondary to metastatic malignant spread. Which
of the following is most likely to be the primary site of malignancy? [B4 Q12]
a. renal
b. breast
c. cervical
d. colon
e. bronchus
Renal
The common cancers that typically metastasize to bone are breast, lung, thyroid, renal and
prostate. Due to the high prevalence of colon cancer, even though only a relatively small
proportion metastasizes to bone, it forms a significant proportion of bone metastases. Prostatic
metastases are typically sclerotic, whereas breast deposits are mixed. Colonic bone metastases
are usually lytic, with renal metastases typically lytic and expansile due to their highly vascular
nature. Other less frequent sources of lytic expansile metastases include thyroid, melanoma
and phaeochromocytoma.
A 35-year-old male presents with a history of backache. Plain radiograph demonstrates
reduction in the lumbar L2/3-disc space with mild endplate irregularity. An MRI of lumbar
spine is carried out for further assessment. What feature on MRI is useful in differentiating
discitis from modic type I endplate change? [B1 Q31]
A. Reduction in the disc height.
B. Low signal change in the endplate on T1WI.
C. High signal change in the endplate on T2WI.
D. Mild irregularity of the endplates.
E. High signal within the disc on T2WI.
High signal within the disc on T2WI.
Degenerative disc disease with associated degenerative modic type 1 endplate change (endplate
oedema) can mimic discitis. All the mentioned changes are seen in both conditions except high
signal within the disc on T2WI, which is seen in infection/discitis. In contrast, the degenerated
disc is of low signal due to loss of hydration.
In addition, disc enhancement and paravertebral inflammatory tissue, soft-tissue mass, and fluid
collection are associated with infection.
A 56-year-old male is admitted under the orthopaedic team with increasingly severe lower back
pain which started three weeks ago. MRI demonstrates an oedematous L4/5 intervertebral disc,
marked loss of disc material and oedematous adjacent endplate changes. There is associated
paravertebral inflammatory tissue and a small amount of pus within the residual disc space.
The findings are consistent with infective discitis. What is the most likely causative organism?
[B2 Q 33]
a. Mycobacterium tuberculosis
b. Streptococcus pyogenes
c. Staphylococcus aureus
d. Escherichia coli
e. Salmonella
Staphylococcus aureus
The most common cause of infective discitis is Staphylococcus aureus, which gives the above
typical findings. The only other relatively common cause is Mycobacterium tuberculosis,
which typically spares the disc until late and usually has a large amount of associated pus.
A 65-year-old Asian man with a long history of back pain is investigated with MRI. Which is
a feature of tuberculosis rather than pyogenic vertebral collapse? [B3 Q10]
A. Rapid progression
B. Marked osteoblastic response
C. Less sclerosis
D. Less collapse
E. Small or no paravertebral abscess
C.
Other features of tuberculosis include slow progression, marked collapse and a large
paravertebral abscess, with or without calcification.
A 16-year-old girl who has recently moved from India to the UK has back pain. Considering
tuberculous spondylitis: [B3 Q30]
A. 10% of skeletal TB involves the spine
B. Infection usually begins in the posterior part of the vertebral body
C. Medial bowing of the psoas shadow on plain film may indicate an abscess
D. The upper thoracic spine is most affected
E. Calcification within a psoas abscess is highly likely to represent TB
Calcification within a psoas abscess is highly likely to represent TB
50% of skeletal TB involves the spine, with the lower thoracic and upper lumbar regions being
most affected. It usually begins at the anterior vertebral body. A psoas abscess may cause lateral
bowing of the psoas shadow on plain film.